Silvestri1501-1600

  • Uploaded by: Linda Kuglarz
  • 0
  • 0
  • October 2019
  • PDF

This document was uploaded by user and they confirmed that they have the permission to share it. If you are author or own the copyright of this book, please report to us by using this DMCA report form. Report DMCA


Overview

Download & View Silvestri1501-1600 as PDF for free.

More details

  • Words: 23,669
  • Pages: 57
PN~CD~Questions~1501-1600-

1

PN Comprehensive Review CD Questions 1501-1600 {No formulas; questions 1506, 1542 (Note to developer: For both s, students will need to use a drag and drop feature to place the lists in the correct order; the correct order is noted in the answers.} 1501. A nurse is reviewing the record of a client admitted to the hospital for treatment of bladder cancer. Which risk factor related to this type of cancer would the nurse most likely note in the client’s record? 1. Female African American 2. Recorded age of 35 years 3. Occupation of computer analyzer 4. Drinks a pot of coffee every day Answer: 4 Rationale: The incidence of bladder cancer is three times greater in men than in women and affects the Caucasian population twice as often as African Americans. It most often occurs after the age of 40 years. Environmental health hazards have been attributed as a cause. Cigarette smoking and drinking coffee are some factors associated with bladder cancer. Test-Taking Strategy: Use the process of elimination. Basic information regarding the general risk factors associated with cancer will direct you to option 4. If you had difficulty with this question, review these risks. Level of Cognitive Ability: Analysis Client Needs: Health Promotion and Maintenance Integrated Process: Nursing Process/Data Collection Content Area: Adult Health/Oncology Reference: Black, J., & Hawks, J. (2005). Medical-surgical nursing: Clinical management for positive outcomes (7th ed.). Philadelphia: W.B. Saunders, p. 867. 1502. A nurse is obtaining data from a client admitted with bladder cancer. Which question would the nurse ask the client to determine if the client experienced the most common symptom associated with this type of cancer? 1. “Do you have frequency with urination?” 2. “Do you commonly feel the urge to urinate?” 3. “Do you experience any pain when you urinate?” 4. “Do you notice any blood in the urine?” Answer: 4 Rationale: The most common symptom in clients with cancer of the bladder is hematuria. The client may also experience irritative voiding symptoms, such as frequency, urgency, and dysuria, and these symptoms are often associated with cancer in situ. The nurse’s question in option 4 will elicit information from the client regarding the most common symptom associated with bladder cancer. Test-Taking Strategy: Note the key words most common. Options 1, 2, and 3 relate to

PN~CD~Questions~1501-1600-

2

obtaining information regarding symptoms that are associated with bladder infection. Review the clinical manifestations associated with bladder cancer if you had difficulty with this question. Level of Cognitive Ability: Analysis Client Needs: Physiological Integrity Integrated Process: Nursing Process/Data Collection Content Area: Adult Health/Oncology Reference: Linton, A., & Maebius, N. (2003). Introduction to medical-surgical nursing (3rd ed.). Philadelphia: W.B. Saunders, p. 778. 1503. A nurse is preparing a client for an intravesical instillation of an alkylating chemotherapeutic agent into the bladder for the treatment of bladder cancer. The nurse provides instructions to the client regarding the procedure. Which statement by the client indicates an understanding of this procedure? 1. “I will need to immediately urinate after the instillation is done.” 2. “I need to stay on bed rest after the procedure is completed.” 3. “After the instillation is done, I will need to retain the fluid for 30 minutes.” 4. “After the instillation is done, I will need to change position every 15 minutes from side to side.” Answer: 4 Rationale: Normally the medication is injected into the bladder through a urethral catheter, the catheter is clamped or removed, and the client is asked to retain the fluid for 2 hours. The client is to change position every 15 to 30 minutes from side to side, and from supine to prone, or to resume all activity immediately during this time period. The client then voids and is instructed to drink water to flush the bladder. Test-Taking Strategy: Knowledge regarding this procedure and posttreatment care is required to answer this question. If you are unfamiliar with this treatment measure, review this content. Level of Cognitive Ability: Analysis Client Needs: Physiological Integrity Integrated Process: Nursing Process/Evaluation Content Area: Adult Health/Oncology Reference: Black, J., & Hawks, J. (2005). Medical-surgical nursing: Clinical management for positive outcomes (7th ed.). Philadelphia: W.B. Saunders, p. 869. 1504. {PLACE FIGURE HERE (Fig. 12)} Perry, A., & Potter, P. (2002). Clinical nursing skills & techniques (5th ed.). St. Louis: Mosby, p. 869. A nurse is providing care to a client with this type of cast. The nurse documents that the client is in a: 1. Short leg cast 2. Long leg cast 3. Body jacket cast 4. Hip spica cast Answer: 4

PN~CD~Questions~1501-1600-

3

Rationale: A hip spica cast is used to treat pelvic and femoral fractures. The cast covers the lower torso and extends to one or both lower extremities. If only one lower extremity is included, it is called a single hip spica; if two are included, it is called a double hip spica. Short and long leg casts are applied to the leg. A body jacket cast is applied to the upper torso. Test-Taking Strategy: Use the process of elimination. Noting the body area covered by the cast will direct you to option 4. Review the various types of casts if you had difficulty with this question. Level of Cognitive Ability: Application Client Needs: Physiological Integrity Integrated Process: Communication and Documentation Content Area: Fundamental Skills Reference: Perry, A., & Potter, P. (2002). Clinical nursing skills & techniques (5th ed.). St. Louis: Mosby, p. 869. 1505. A nurse is developing a plan of care for a client following a radical mastectomy and includes measures that will assist in preventing lymphedema of the affected arm. The nurse should include which of the following to prevent this complication? 1. Place the affected arm in a dependent position 2. Place a cool compress on the affected arm 3. Place the affected arm on a pillow 4. Instruct the client to avoid simple arm exercises in the affected arm Answer: 3 Rationale: Following mastectomy, the arm should be elevated above the level of the heart. Simple arm exercises should be encouraged. No blood pressure readings, injections, IV lines, or blood draws should be performed on the affected arm. Cool compresses are not a suggested measure to prevent lymphedema from occurring. Test-Taking Strategy: Note the key words prevent this complication. Use the process of elimination and knowledge regarding the pathophysiology associated with lymphedema to answer the question. Review these measures if you had difficulty with this question. Level of Cognitive Ability: Application Client Needs: Physiological Integrity Integrated Process: Nursing Process/Planning Content Area: Adult Health/Oncology Reference: Christensen, B., & Kockrow, E. (2003). Adult health nursing (4th ed.). St. Louis: Mosby, p. 534. 1506. A nurse is assigned to care for three clients on the day shift. List the order in which the nurse should care for the clients. A client who is scheduled for a chest x-ray at 12 PM. A client requiring an abdominal wound irrigation to be performed three times daily with the first irrigation to be done in the morning A client being discharged from the hospital this morning who will need reinforcement about performing activities of daily living Answer: A client requiring an abdominal wound irrigation to be performed three times daily with

PN~CD~Questions~1501-1600-

4

the first irrigation to be done in the morning A client being discharged from the hospital this morning who will need reinforcement about performing activities of daily living A client who is scheduled for a chest x-ray at 12 PM. Rationale: Since the client requiring wound irrigations needs to have the irrigations performed three times daily, the client will need to be cared for early in the day and have the wound irrigation performed early. Since the client requiring a chest x-ray is scheduled at 12 PM, this client can be cared for last. The client being discharged from the hospital should be cared for second since this client requires reinforcement about performing activities of daily living. Test-Taking Strategy: Think about the needs of each client in determining the correct option. Noting that the abdominal wound irrigation needs to be performed three times daily will assist in determining that this procedure needs to be performed first. Also noting that the client requiring a chest x-ray is scheduled at 12 PM. will assist in determining that this client can be cared for last. If you had difficulty with this question, review time management and prioritization of client needs. Level of Cognitive Ability: Application Client Needs: Safe, Effective Care Environment Integrated Process: Nursing Process/Planning Content Area: Delegating/Prioritizing Reference: Potter, P., & Perry, A. (2005). Fundamentals of nursing (6th ed.). St. Louis: Mosby, pp. 377-378. 1507. A nurse in charge of a rehabilitation center is planning the client assignments for the day. Which of the following clients would the nurse assign to the nursing assistant? 1. A client admitted to the center yesterday who had a below-the-knee amputation 2. A client on a 24-hour urine collection who is on strict bed rest 3. A client scheduled to be discharged home following rehabilitation for a hip replacement procedure 4. A client scheduled to be transferred to the hospital for a cardiac catheterization Answer: 2 Rationale: The nurse is legally responsible for client assignments and must assign tasks based on the guidelines of nursing practice acts and the job description of the employing agency. A newly admitted client who had a below-the-knee amputation will require both physiological and psychosocial care and initiation of rehabilitation. A client scheduled to be discharged home will require reinforcement of home care management. A client scheduled for a cardiac catheterization requires physiological needs. The nursing assistant has been trained to care for a client on bed rest and on urine collections. The nurse would provide instructions to the nursing assistant regarding the tasks, but the tasks required for this client are within the role description of a nursing assistant. Test-Taking Strategy: Note that the question asks for the assignment to be delegated to the nursing assistant. When asked questions related to delegation, think about the role description of the employee and the needs of the client. By the process of elimination, you will easily be directed to option 2. Review the responsibilities related to delegation if you had difficulty with this question. Level of Cognitive Ability: Application

PN~CD~Questions~1501-1600-

5

Client Needs: Safe, Effective Care Environment Integrated Process: Nursing Process/Planning Content Area: Delegating/Prioritizing Reference: Potter, P., & Perry, A. (2005). Fundamentals of nursing (6th ed.). St. Louis: Mosby, pp. 42, 378-379, 418. 1508. {PLACE FIGURE HERE (Fig. 13)} Murray, S., McKinney, E., & Gorrie, T., (2002). Foundations of maternalnewborn nursing (3rd ed.). Philadelphia: W.B. Saunders, p. 426. It has been 12 hours since the client’s delivery of a newborn. The nurse assesses the mother for the process of involution and documents that it is progressing normally when palpation of the client’s fundus is noted at which level? Answer: 2 Rationale: The term involution is used to describe the rapid reduction in size and the return of the uterus to a normal condition similar to its pregnant state. Immediately following the delivery of the placenta, the uterus contracts to the size of a large grapefruit. The fundus is situated in the midline between the symphysis pubis and the umbilicus. Within 6 to 12 hours after birth, the fundus of the uterus rises to the level of the umbilicus. The top of the fundus remains at the level of the umbilicus for about a day and then descends into the pelvis approximately one fingerbreadth on each succeeding day. Test-Taking Strategy: Note the key words it has been 12 hours. Visualize the process of assessment of involution and the expected finding at this time to answer the question. Remember that within 6 to 12 hours after birth, the fundus of the uterus rises to the level of the umbilicus remaining at this level for about a day, then descends into the pelvis approximately one fingerbreadth on each succeeding day. Review this process if you had difficulty with this question. Level of Cognitive Ability: Application Client Needs: Health Promotion and Maintenance Integrated Process: Nursing Process/Data Collection Content Area: Maternity/Postpartum Reference: Murray, S., McKinney, E., & Gorrie, T., (2002). Foundations of maternalnewborn nursing (3rd ed.). Philadelphia: W.B. Saunders, p. 426. 1509. A nurse asks a nursing student to describe case management. Which response by the student indicates a lack of understanding about this concept? 1. “It represents a primary health prevention focus managed by a single case manager.” 2. “It is managing client care by managing the client care environment.” 3. “It is designed to promote appropriate use of hospital personnel and material resources.” 4. “It maximizes hospital revenues while providing for optimal outcome of client care.” Answer: 1 Rationale: Case management represents an interdisciplinary health care delivery system to promote appropriate use of hospital personnel and material resources to maximize hospital revenues while providing for optimal outcome of care. Options 2, 3, and 4 identify the components of managed care.

PN~CD~Questions~1501-1600-

6

Test-Taking Strategy: Use the process of elimination. Note the key words lack of understanding. These words indicate a false-response question and that you need to select the incorrect student statement. Recalling the characteristics of case management and noting the word “single” in option 1 will direct you to this option. Review the characteristics of case management if you had difficulty with this question. Level of Cognitive Ability: Analysis Client Needs: Safe, Effective Care Environment Integrated Process: Teaching/Learning Content Area: Leadership/Management Reference: Potter, P., & Perry, A. (2005). Fundamentals of nursing (6th ed.). St. Louis: Mosby, p. 373. 1510. A nurse is assisting in reviewing the critical paths of the clients on the nursing unit. In performing a variance analysis, which of the following would indicate a negative variance? 1. Signs of wound healing in a postoperative abdominal incision 2. The presence of dysrhythmias in a client with a myocardial infarction 3. Normal vital signs in a postoperative craniotomy client 4. A client demonstrating accurate insulin administration following teaching Answer: 2 Rationale: Variances are actual deviations or detours from the critical paths. Variances can be either positive or negative, avoidable or unavoidable, and can be caused by a variety of things. Positive variance occurs when the client achieves maximal benefit and is discharged earlier than anticipated. Negative variance occurs when untoward events prevent a timely discharge. Variance analysis occurs continually to anticipate and recognize negative variance early so that appropriate action can be taken. Test-Taking Strategy: Use the process of elimination, identifying the negative variance. Options 1, 3, and 4 identify positive outcomes. Option 2 identifies a negative outcome. Review the purpose of variance analysis if you had difficulty with this question. Level of Cognitive Ability: Analysis Client Needs: Safe, Effective Care Environment Integrated Process: Nursing Process/Evaluation Content Area: Leadership/Management Reference: Potter, P., & Perry, A. (2005). Fundamentals of nursing (6th ed.). St. Louis: Mosby, pp. 485-487. 1511. A nurse is reviewing the nursing care plan prepared by a nursing student who is preparing to care for a client undergoing a pneumonectomy for a lung tumor. Which of the following nursing interventions if documented in the plan of care would the nurse question? 1. Monitor the amount and characteristics of the drainage from the chest tube system 2. Encourage the client to cough and deep breath 3. Keep the head of the bed elevated 4. Assess the surgical dressing for drainage Answer: 1 Rationale: Closed chest drainage is not usually used following pneumonectomy. The

PN~CD~Questions~1501-1600-

7

serous fluid that accumulates in the empty thoracic cavity eventually consolidates. The consolidation prevents shifts of the mediastinum, heart, and remaining lung. The head of the bed is elevated to facilitate adequate oxygenation. Complete lateral positioning is avoided because the mediastinum is no longer held in place on both sides by lung tissue, and extreme turning may cause mediastinal shift and compression of the remaining lung. Options 2 and 4 are general postoperative measures. Test-Taking Strategy: Note the key words would the nurse question. These words indicate a false-response question and that you need to select the incorrect intervention. Use the process of elimination and eliminate options 2 and 4 first because they are general postoperative measures. Attempt to visualize the effects of the surgical procedure in selecting the correct option. Recalling that in a pneumonectomy the entire lung is removed will assist in directing you to option 1. If you had difficulty with this question, review postoperative care of this surgical procedure. Level of Cognitive Ability: Analysis Client Needs: Physiological Integrity Integrated Process: Teaching/Learning Content Area: Adult Health/Oncology Reference: Christensen, B., & Kockrow, E. (2003). Adult health nursing (4th ed.). St. Louis: Mosby, p. 391. 1512. A nurse is attending an agency orientation regarding the nursing model of practice implemented in the facility. The nurse is told that the nursing model is a team nursing approach. The nurse determines that which of the following is a characteristic of this type of nursing model of practice? 1. A task approach method is used to provide care to clients. 2. A single registered nurse (RN) is responsible for providing nursing care to a group of clients 3. Managed care concepts and tools are used in providing client care 4. Nursing personnel are led by an RN leader in providing care to a group of clients Answer: 4 Rationale: In team nursing, nursing personnel are led by an RN leader to provide care to a group of clients. Option 1 identifies functional nursing. Option 2 identifies primary nursing. Option 3 identifies a component of case management. Test-Taking Strategy: Note that the issue of the question relates to team nursing. Keep this issue in mind and use the process of elimination. Option 4 is the only option that identifies the concept of a team approach. Review the various types of nursing delivery systems if you had difficulty with this question. Level of Cognitive Ability: Analysis Client Needs: Safe, Effective Care Environment Integrated Process: Nursing Process/Evaluation Content Area: Leadership/Management Reference: Potter, P., & Perry, A. (2005). Fundamentals of nursing (6th ed.). St. Louis: Mosby, p. 373. 1513. A nurse is assigned to assist with working with food services in a rural, poor school setting. A goal for the school dietary program is to prevent nutritional deficiencies

PN~CD~Questions~1501-1600-

8

and enhance children’s nutritional status through healthy dietary practices. In implementing interventions by levels of prevention, which of the following would be a primary prevention intervention that the nurse could suggest to use? 1. Case finding in the school to identify dietary practices 2. School screening programs for early detection of children with poor eating habits 3. Providing educational programs, literature, and posters to promote awareness of healthy eating 4. Conduct a community-wide dietary screening activity to detect community dietary trends Answer: 3 Rationale: Primary prevention interventions are those measures that keep illness, injury, or potential problems from occurring. Options 1, 2, and 4 are secondary prevention measures that seek to detect existing health problems or trends. Test-Taking Strategy: Note the issue of the question “primary prevention intervention.” Knowledge that primary prevention interventions are those measures that keep illness from occurring will direct you to option 3. If you had difficulty with this question, review the levels of prevention. Level of Cognitive Ability: Application Client Needs: Safe, Effective Care Environment Integrated Process: Nursing Process/Implementation Content Area: Leadership/Management Reference: Potter, P., & Perry, A. (2005). Fundamentals of nursing (6th ed.). St. Louis: Mosby, pp. 349, 784. 1514. A nurse is assisting with working with disaster relief following a tornado. The nurse’s goal with the overall community is to prevent as much injury and death as possible from the uncontrollable event. Finding safe housing for survivors, providing support to families, organizing counseling sessions, and securing physical care when needed are all examples of which type of prevention? 1. The primary level of prevention 2. The secondary level of prevention 3. The tertiary level of prevention 4. Aggregate care prevention Answer: 3 Rationale: Tertiary prevention involves the reduction of the amount and degree of disability, injury, and damage following a crisis. Primary prevention means keeping the crisis from ever occurring, and secondary prevention focuses on reducing the intensity and duration of the crisis during the crisis itself. There is no known aggregate care prevention level. Test-Taking Strategy: Identify the information in the question. Focus on this information and use knowledge regarding the various levels of prevention to answer the question. Remember that tertiary prevention involves the reduction of the amount and degree of disability, injury, and damage following a crisis. If you had difficulty with this question, review the levels of prevention. Level of Cognitive Ability: Application Client Needs: Safe, Effective Care Environment

PN~CD~Questions~1501-1600-

9

Integrated Process: Nursing Process/Implementation Content Area: Leadership/Management References: Potter, P., & Perry, A. (2005). Fundamentals of nursing (6th ed.). St. Louis: Mosby, p. 97. Stuart, G., & Laraia, M. (2005). Principles & practice of psychiatric nursing (8th ed.). St. Louis: Mosby, pp. 209, 239. 1515. A nurse is assisting in planning the client assignments. Which of the following is the least appropriate assignment for the nursing assistant? 1. Assisting a profoundly developmentally disabled child to eat lunch 2. Obtaining frequent oral temperatures on a client 3. Accompanying a 51-year-old man, being discharged to home following a bowel resection 8 days ago, to his transportation 4. Collecting a urine specimen from a 70-year-old woman admitted 3 days ago Answer: 1 Rationale: The nurse must determine the most appropriate assignment based on the skills of the staff member and the needs of the client. In this case the least appropriate assignment for a nursing assistant would be assisting with feeding a profoundly developmentally disabled child. The child is likely to have difficulty eating, and therefore a higher potential for complications, such as choking and aspiration exists. The remaining three options include no data that indicates that these tasks carry any unforeseen risk. Test-Taking Strategy: Note the key words least appropriate. These words indicate a false-response question and that you need to select the incorrect assignment. Use the ABCs—airway, breathing, and circulation—and recall the principles of delegation and supervision of tasks to others in answering the question. Remember that work that is delegated to others must be done consistent with the individual’s level of expertise and licensure or lack of licensure. Review the principles of assignments and delegation if you had difficulty with this question. Level of Cognitive Ability: Application Client Needs: Safe, Effective Care Environment Integrated Process: Nursing Process/Planning Content Area: Delegating/Prioritizing Reference: Potter, P., & Perry, A. (2005). Fundamentals of nursing (6th ed.). St. Louis: Mosby, pp. 42, 378-379, 418. 1516. A nurse is assigned to care for four clients. In planning client rounds, which client would the nurse assess first? 1. A client receiving oxygen via nasal cannula who had difficulty breathing during the previous shift 2. A postoperative client preparing for discharge 3. A client scheduled for a chest x-ray 4. A client requiring daily dressing changes Answer: 1 Rationale: Airway is always a high priority, and the nurse would attend to the client who has been experiencing an airway problem first. The clients described in options 2, 3, and

PN~CD~Questions~1501-1600-

10

4 would be an intermediate priority. Test-Taking Strategy: Use Maslow’s Hierarchy of Needs theory and the ABCs—airway, breathing, and circulation—to answer the question. Remember that airway is always the first priority. Review the guidelines related to prioritizing if you had difficulty with this question. Level of Cognitive Ability: Application Client Needs: Safe, Effective Care Environment Integrated Process: Nursing Process/Planning Content Area: Delegating/Prioritizing Reference: Potter, P., & Perry, A. (2005). Fundamentals of nursing (6th ed.). St. Louis: Mosby, pp. 318-319. 1517. A nurse has provided instructions to a client scheduled for a mammography regarding the procedure. Which statement by the client indicates an understanding of the procedure? 1. “The test takes about 1 hour and is painless.” 2. “I cannot wear deodorant on the day of the test.” 3. “I will need to wear a sports bra for the procedure.” 4. “I cannot eat on the day of the test.” Answer: 2 Rationale: Mammography takes about 15 to 30 minutes to complete. Some discomfort may be experienced because of the breast compression required to obtain a clear image. There is no reason to maintain an NPO status before the procedure. There is also no reason to wear a sports bra. The test is performed without clothing. Deodorants, powders, and lotions cannot be worn on the day of the test because it will affect the testing process and affect the imaging of the breasts. Test-Taking Strategy: Note the key words indicates an understanding. Visualize the procedure to assist in selecting the correct option. Eliminate options 3 and 4 first. Knowing that the test causes some discomfort will assist in eliminating option 1. If you are unfamiliar with this screening test, review this content. Level of Cognitive Ability: Analysis Client Needs: Health Promotion and Maintenance Integrated Process: Teaching/Learning Content Area: Adult Health/Oncology Reference: Chernecky, C., & Berger, B. (2004). Laboratory tests and diagnostic procedures (4th ed.). Philadelphia: W.B. Saunders, p. 765. 1518. A nurse informs a client that a Papanicolaou smear will be done at the next scheduled clinic visit, and the nurse provides instructions to the client regarding preparation for this test. Which statement by the client indicates an understanding of the procedure? 1. “If I have my period at the time of my next scheduled visit, I will not be able to have the test done.” 2. “I need to restrict fluids on the day of the test.” 3. “The test is painful and will cause cramping.” 4. “I need to do a vaginal douching with vinegar and water on the morning of my

PN~CD~Questions~1501-1600-

11

appointment.” Answer: 1 Rationale: A Papanicolaou smear cannot be performed during menstruation. The test is usually painless. The client needs to be instructed to avoid douching for at least 24 hours before the test. There is no reason to restrict fluids on the day of the test. Test-Taking Strategy: Knowledge regarding the Papanicolaou test is required to answer the question. Eliminate option 2 first as an unlikely preparation measure, although the client may be asked to urinate before the test. Eliminate option 4 next because douching will affect the results of the test. Knowing that the test is painless will direct you to option 1 as the answer to the question. Review client preparation for a Papanicolaou test if you had difficulty with this question. Level of Cognitive Ability: Analysis Client Needs: Health Promotion and Maintenance Integrated Process: Teaching/Learning Content Area: Adult Health/Oncology References: Black, J., & Hawks, J. (2005). Medical-surgical nursing: Clinical management for positive outcomes (7th ed.). Philadelphia: W.B. Saunders, pp. 998, 1074. Chernecky, C., & Berger, B. (2004). Laboratory tests and diagnostic procedures (4th ed.). Philadelphia: W.B. Saunders, p. 840. Lewis, S., Heitkemper, M., & Dirksen, S. (2004). Medical-surgical nursing: Assessment and management of clinical problems (6th ed.). St. Louis: Mosby, p. 1356. 1519. A nurse assists in caring for a client with an inoperable lung tumor and assists in developing a plan of care addressing complications related to the disorder. The nurse includes in the plan to monitor for the early signs of superior vena cava syndrome. Which of the following would the nurse include in the plan as the early sign of this oncological emergency? 1. Hand and arm edema 2. Signs of disorientation 3. Edema of the face and eyes 4. Bluish skin discoloration around the mouth Answer: 3 Rationale: Superior vena cava syndrome occurs when the superior vena cava is compressed or obstructed by tumor growth. Early signs and symptoms generally occur in the morning and include edema of the face, especially around the eyes, and client complaints of tightness of a shirt or blouse collar. As the compression worsens, the client experiences edema of the hands and arms. Mental status changes and cyanosis are late signs. Test-Taking Strategy: Note the key word early in the stem of the question. This key word should assist in eliminating options 1, 2, and 4. If you are unfamiliar with superior vena cava syndrome, review this oncological emergency. Level of Cognitive Ability: Application Client Needs: Physiological Integrity Integrated Process: Nursing Process/Planning Content Area: Adult Health/Oncology

PN~CD~Questions~1501-1600-

12

Reference: Black, J., & Hawks, J. (2005). Medical-surgical nursing: Clinical management for positive outcomes (7th ed.). Philadelphia: W.B. Saunders, p. 1854. 1520. A client has a chest tube that is attached to a Pleur-Evac drainage system. The client asks the nurse, “Can the tube come out faster if you turn the wall suction up higher?” The nurse’s response is based on the understanding that turning up the wall suction would: 1. Not increase the actual suction in the system, but would cause more air to be pulled through the air vent and suction chamber to the suction source 2. Not increase the actual suction in the system, but could cause the client to suffer injury 3. Increase the actual suction in the system, but could damage lung tissue 4. Increase the actual suction in the system and is a good idea Answer: 1 Rationale: The amount of suction in the Pleur-Evac drainage system is controlled by the amount of sterile water that is poured into the suction control chamber. In a dry suction system, this is accomplished by regulating the suction dial on the Pleur-Evac. Increasing the wall suction will only cause vigorous bubbling in the suction chamber because more air is pulled through the air vent and suction control chamber to the suction source. The only effect this would have is to increase the rate of water evaporation from the suction control chamber, so sterile water would have to be added to the system more frequently. Test-Taking Strategy: Use the process of elimination. Basic knowledge of the functioning of the chambers of a Pleur-Evac chest drainage system is necessary to answer this question. Knowing that it is the water level in the chamber that regulates the amount of suction (not the level of the wall suction control) helps you to eliminate options 3 and 4. From the remaining options, recalling that the only effect of added wall suction is the evaporation of the water at a faster rate will direct you to option 1. If you had difficulty with this question, review chest tube drainage systems. Level of Cognitive Ability: Analysis Client Needs: Physiological Integrity Integrated Process: Nursing Process/Implementation Content Area: Adult Health/Respiratory Reference: Christensen, B., & Kockrow, E. (2003). Adult health nursing (4th ed.). St. Louis: Mosby, pp. 384-385. 1521. A nurse has reinforced discharge instructions regarding home care to a client following a prostatectomy for cancer of the prostate. Which statement by the client indicates an understanding of the instructions? 1. “I should not lift anything more than 20 lb.” 2. “To prevent dribbling of urine, I need to limit my fluid intake to four glasses daily.” 3. “If I see any clots in my urine, I need to call the physician immediately.” 4. “I can begin to drive my car in 1 week.” Answer: 1 Rationale: The client needs to be instructed to avoid lifting objects heavier than 20 lb for at least 6 weeks. Small pieces of tissue or blood clots can be passed during urination for up to 2 weeks after surgery and if noticed does not necessitate the need to notify the

PN~CD~Questions~1501-1600-

13

physician. Driving a car and sitting for long periods of time are restricted for at least 3 weeks. A high daily fluid intake of 2 to 2.5 L/day should be maintained to limit clot formation and prevent infection. Test-Taking Strategy: Use the process of elimination noting the key words an understanding of the instructions. Option 2 can be eliminated first. Eliminate option 4 next because 1 week is a rather short time period. Recalling that blood clots are expected following this type of surgery will assist in directing you to option 1. Review client teaching points following prostatectomy if you had difficulty with this question. Level of Cognitive Ability: Analysis Client Needs: Health Promotion and Maintenance Integrated Process: Nursing Process/Evaluation Content Area: Adult Health/Oncology Reference: Linton, A., & Maebius, N. (2003). Introduction to medical-surgical nursing (3rd ed.). Philadelphia: W.B. Saunders, p. 982. 1522. A nurse is planning the client assignments for the shift. Which of the following clients would the nurse most appropriately assign to the nursing assistant? 1. A client requiring twice daily dry dressing changes 2. A client requiring frequent ambulation with a walker 3. A client on a bowel management program requiring rectal suppositories and a daily enema 4. A client with diabetes mellitus requiring daily insulin and reinforcement of dietary measures Answer: 2 Rationale: Assignment of tasks needs to be implemented based on the job description of the nursing assistant, the level of clinical competence, and state law. Options 1, 3, and 4 involve care that requires the skill of a licensed nurse. Although a nursing assistant may be trained to administer an enema (depending on the state practice act and agency policies), a rectal suppository needs to be administered by a licensed nurse. Option 2 is the most appropriate assignment for the nursing assistant. Test-Taking Strategy: Use the process of elimination and knowledge regarding tasks that can be safely delegated to the nursing assistant. Eliminate options 1, 3, and 4 because these clients require care or a component of care that needs to be provided by the licensed nurse. A nursing assistant is trained to ambulate a client with a walker. Review the principles related to delegation and assignment making if you had difficulty with this question. Level of Cognitive Ability: Application Client Needs: Safe, Effective Care Environment Integrated Process: Nursing Process/Planning Content Area: Delegating/Prioritizing Reference: Potter, P., & Perry, A. (2005). Fundamentals of nursing (6th ed.). St. Louis: Mosby, pp. 42, 378-379, 418. 1523. A student nurse has received the client assignment for the day and is organizing the required tasks. The nursing instructor reviews the plan for time management with the student and determines that the student needs assistance with the plan if the student

PN~CD~Questions~1501-1600-

14

indicated that which activity should be part of it? 1. Prioritizing client needs and daily tasks 2. Providing time for unexpected tasks 3. Gathering supplies before beginning a task 4. Documenting task completion at the end of the day Answer: 4 Rationale: The nurse should document task completion continuously throughout the day. Options 1, 2, and 3 identify accurate components of time management. Test-Taking Strategy: Use the process of elimination and knowledge regarding the guidelines for time management to answer the question. Note the key words student needs assistance. These words indicate a false-response question and that you need to select the incorrect student statement. If you had difficulty with this question, review time management principles and the principles related to documentation. Level of Cognitive Ability: Comprehension Client Needs: Safe, Effective Care Environment Integrated Process: Teaching/Learning Content Area: Leadership/Management Reference: Potter, P., & Perry, A. (2005). Fundamentals of nursing (6th ed.). St. Louis: Mosby, pp. 378, 481-483. 1524. A nurse is giving a bed bath to an assigned client. A nursing assistant enters the client’s room and tells the nurse that another assigned client is in pain and needs pain medication. The nurse should do which of the following? 1. Finish the bed bath and then administer the pain medication to the other client 2. Cover the client, raise the side rails, tell the client that you will return shortly, and administer the pain medication to the other client 3. Ask the nursing assistant to tell the client in pain that medication will be administered as soon as the bed bath is complete 4. Ask the nursing assistant to find out when the last pain medication was given to the client Answer: 2 Rationale: The nurse is responsible for the care provided to the assigned clients. The most appropriate action is to provide safety to the client that is receiving the bed bath and prepare to administer the pain medication. Options 1 and 3 delay the administration of medication to the client in pain. Option 4 is not a responsibility of the nursing assistant. Test-Taking Strategy: Use the process of elimination and principles related to priorities of care. Options 1 and 3 delay the administration of pain medication, and option 4 is not a responsibility of the nursing assistant. The appropriate action is to administer the medication. Review the guidelines related to prioritizing if you had difficulty with this question. Level of Cognitive Ability: Application Client Needs: Safe, Effective Care Environment Integrated Process: Nursing Process/Implementation Content Area: Leadership/Management Reference: Potter, P., & Perry, A. (2005). Fundamentals of nursing (6th ed.). St. Louis: Mosby, pp. 267, 318-320.

PN~CD~Questions~1501-1600-

15

1525. A new nurse is employed at a local community hospital and is attending an orientation session. The nurse educator conducting the session asks the new nurse to describe an organization’s mission statement. The new nurse most appropriately responds by telling the nurse educator that it: 1. Defines the rules of the organization that the employees must follow 2. Identifies the policies and procedures of the organization 3. Outlines what the organization plans to accomplish 4. Describes the benefits available to employees Answer: 3 Rationale: All organizations have a purpose or reason for existing. The purpose is typically expressed in the form of a mission statement. The mission statement outlines what the organization plans to accomplish. Sometimes mission statements incorporate statements of philosophy (beliefs), purpose, and goals or objectives into a single statement; at other times, the philosophy, purposes, and goals are addressed in addition to the mission statement. These statements serve as a benchmark against which an organization’s performance can be evaluated. Test-Talking Strategy: Use the process of elimination focusing on the issue, mission statement. Note the relationship between the definition of a mission and option 3. Review the description of an organization’s mission statement if you had difficulty with this question. Level of Cognitive Ability: Application Client Needs: Safe, Effective Care Environment Integrated Process: Teaching/Learning Content Area: Leadership/Management Reference: Potter, P., & Perry, A. (2005). Fundamentals of nursing (6th ed.). St. Louis: Mosby, p. 372. 1526. A nurse is newly employed in a health agency. The nurse is told that the decisionmaking process of the organization is based on a centralized structure. The nurse determines that this means that the authority to make decisions is vested in: 1. Each employee 2. A few individuals, such as the board of directors 3. All members of the organization 4. Many individuals filtering down to the individual employee Answer: 2 Rationale: With regard to the decision-making process, organizations may be described as having a centralized or decentralized structure. An organization is depicted as centralized when the authority to make decisions is vested in a few individuals. Conversely, when the decision making involves a number of individuals and filters down to the individual employee, the organization is said to operate in a decentralized fashion. Test-Taking Strategy: Use the process of elimination. Note that options 1, 3, and 4 are similar in that they address all employees of the organization. Review the differences between a centralized and decentralized organization if you had difficulty with this question. Level of Cognitive Ability: Comprehension

PN~CD~Questions~1501-1600-

16

Client Needs: Safe, Effective Care Environment Integrated Process: Nursing Process/Implementation Content Area: Leadership/Management Reference: Potter, P., & Perry, A. (2005). Fundamentals of nursing (6th ed.). St. Louis: Mosby, p. 374. 1527. A nurse is reinforcing instructions to a client following mastectomy who will be discharged with an axillary drain in place. The client will be receiving home care visits from a registered nurse to monitor drainage and perform dressing changes. Which statement by the client indicates a need for further instructions? 1. “I need to begin full range-of-motion (ROM) exercises to my upper arm as soon as I get home.” 2. “I need to keep my arm elevated when I sit or lie down.” 3. “I may feel pain in the breast area even though my breast has been removed.” 4. “I can massage the area with cocoa butter once the incision heals.” Answer: 1 Rationale: The client should be instructed to limit upper arm ROM to the level of the shoulder only. After the axillary drain is removed, the client can begin full ROM exercises to the upper arm as prescribed. Options 2, 3, and 4 are correct measures following a mastectomy. Test-Taking Strategy: Use the process of elimination and focus on the issue that the client is being discharged with an axillary drain in place. Note the key words need for further instructions. These words indicate a false-response question and that you need to select the incorrect client statement. Noting the word “full” in option 1 will direct you to this option. If you had difficulty with this question, review the client teaching points following mastectomy. Level of Cognitive Ability: Comprehension Client Needs: Health Promotion and Maintenance Integrated Process: Teaching/Learning Content Area: Adult Health/Oncology Reference: Linton, A., & Maebius, N. (2003). Introduction to medical-surgical nursing (3rd ed.). Philadelphia: W.B. Saunders, pp. 958-959. 1528. A nurse assists in developing a plan of care for a client who will be hospitalized for insertion of an internal cervical (vaginal) radiation implant. Which of the following should the nurse suggest including in the plan of care for the client? 1. Place the client in a private room close to the nurses’ station 2. Place a radiation sign on the door of the client’s room 3. Reinsert the implant into the vagina immediately if it becomes dislodged 4. Limit visitors’ time to 60-minute visits Answer: 2 Rationale: The client’s room should be marked with appropriate signs stating the presence of radiation. The client should be placed in a private room at the end of the hall because this location provides less of a chance of exposure of radiation to others. A lead container and long-handled forceps should be kept in the client’s room at all times during internal radiation therapy. If the implant becomes dislodged, the nurse should pick up the

PN~CD~Questions~1501-1600-

17

implant with long-handled forceps and place it in the lead container. It is not reinserted by the nurse. The nurse should limit visitors’ time to 30-minute visits. Test-Taking Strategy: Use the process of elimination and knowledge regarding the precaution and care to a client with a radiation implant to answer the question. Eliminate option 4 because of the lengthy time frame for visits. Eliminate option 1 because of the words “close to the nurses’ station.” Knowing that it is not within the scope of nursing practice to reinsert an implant will assist in eliminating option 3. Review these precautions if you had difficulty with this question. Level of Cognitive Ability: Application Client Needs: Safe, Effective Care Environment Integrated Process: Nursing Process/Planning Content Area: Adult Health/Oncology Reference: Black, J., & Hawks, J. (2005). Medical-surgical nursing: Clinical management for positive outcomes (7th ed.). Philadelphia: W.B. Saunders, p. 363. 1529. A nurse is assisting in caring for a client receiving chemotherapy. On review of the morning laboratory results, the nurse notes that the white blood cell count is extremely low, and the client is immediately placed on neutropenic precautions. The client’s breakfast tray arrives, and the nurse inspects the meal and prepares to bring the tray into the client’s room. Which of the following actions should the nurse take before bringing the meal to the client? 1. Remove the fresh orange off of the breakfast tray 2. Remove the coffee from the breakfast tray 3. Call the dietary department and ask for disposable utensils 4. Ask the client if she feels like eating at this time Answer: 1 Rationale: In the immunocompromised client, a low bacteria diet is implemented. This includes avoiding fresh fruits and vegetables and thorough cooking of all foods. It is not necessary to remove the coffee from the tray. Disposable utensils are used for clients who are infectious and present a risk of transmitting an infection to others. It is best to encourage the client to eat because nutrition is very important in a client receiving chemotherapy who is immunocompromised. Test-Taking Strategy: Focus on the issue of the question, “neutropenic precautions.” Eliminate option 4 because this is not the best measure in the client who requires nutrition. There is no reason for option 2; therefore eliminate this option. Knowing that fresh fruits and vegetables present a threat to this client or knowing that disposable utensils are used for the client that is infectious will direct you to the correct option. Review interventions for the client with hematological toxicity on neutropenic precautions if you had difficulty with this question. Level of Cognitive Ability: Application Client Needs: Safe, Effective Care Environment Integrated Process: Nursing Process/Implementation Content Area: Adult Health/Oncology Reference: Linton, A., & Maebius, N. (2003). Introduction to medical-surgical nursing (3rd ed.). Philadelphia: W.B. Saunders, p. 540.

PN~CD~Questions~1501-1600-

18

1530. A nurse is assisting in caring for a client with a diagnosis of bladder cancer who recently received chemotherapy. The nurse receives a telephone call from the laboratory that reports that the client’s platelet count is 20,000/mm3. Based on this laboratory value, the nurse revises the plan of care and suggests including which intervention? 1. Instruct the client that if anyone delivers fresh flowers that they should be returned to the florist 2. Instruct the client not to eat any fresh fruits 3. Monitor for signs of infection in the client 4. Monitor skin for the presence of petechiae Answer: 4 Rationale: When the platelet count is decreased, the client is at risk for bleeding. A high risk of hemorrhage exists when the platelet count is less than 20,000/mm3. Fatal central nervous system hemorrhage or massive gastrointestinal hemorrhage can occur when the platelet count is less than 10,000/mm3. The client should be assessed for signs of bleeding. Options 2 and 3 are specific interventions related to the risk of infection, and although they may be a component of the plan of care, they are not specific to the risk for bleeding. Contact with fresh flowers is avoided when the client is at risk for infection. Additionally, option 1 is not a therapeutically stated instruction to the client. Test-Taking Strategy: Use the process of elimination. Recalling the normal platelet count and determining that a low count places the client at risk for bleeding will assist in eliminating options 1, 2, and 3. Review the normal platelet count and the nursing interventions for a client with a low count if you had difficulty with this question. Level of Cognitive Ability: Application Client Needs: Physiological Integrity Integrated Process: Nursing Process/Planning Content Area: Adult Health/Oncology Reference: Linton, A., & Maebius, N. (2003). Introduction to medical-surgical nursing (3rd ed.). Philadelphia: W.B. Saunders, pp. 516, 518. 1531. A nurse is employed in a long-term care facility as a charge nurse of the night shift. The nurse determines that in this position of authority, authority most appropriately refers to: 1. Being responsible for what the staff members do 2. Carrying the legal responsibility for the task performance of others 3. The official power to approve an action, command an action, or to see that a decision is enforced 4. Accepting the responsibility for the actions of others Answer: 3 Rationale: Authority refers to the official power an individual has to approve an action, to command an action, or to see that a decision is enforced. Options 1, 2, and 4 are not related to the description of a position of authority. Test-Taking Strategy: Use the process of elimination and knowledge regarding the description of a position of authority. Note the relationship between the word “authority” in the question and “power” in the correct option. Also note that options 1, 2, and 4 are similar. Review the description of authority if you had difficulty with this question.

PN~CD~Questions~1501-1600-

19

Level of Cognitive Ability: Comprehension Client Needs: Safe, Effective Care Environment Integrated Process: Nursing Process/Planning Content Area: Leadership/Management Reference: Potter, P., & Perry, A. (2005). Fundamentals of nursing (6th ed.). St. Louis: Mosby, p. 374. 1532. A nursing instructor asks a nursing student to describe accountability. Which statement by the student indicates an inaccurate description of accountability? 1. “One must answer for the care that one asks others to complete.” 2. “It carries legal implications for task performance.” 3. “It refers to the process of answering or being responsible for what occurs.” 4. “Accountability can be delegated.” Answer: 4 Rationale: Accountability refers to the process of answering or being responsible for what occurs and carries legal implications for task performance. Accountability cannot be delegated; one must answer for the care given and for the care one asks others to complete. Test-Taking Strategy: Use the process of elimination noting the key word inaccurate in the stem of the question. These words indicate a false-response question and that you need to select the incorrect student statement. Recalling the definition of accountability will direct you to option 4. Review this definition if you had difficulty with this question. Level of Cognitive Ability: Comprehension Client Needs: Safe, Effective Care Environment Integrated Process: Teaching/Learning Content Area: Leadership/Management Reference: Potter, P., & Perry, A. (2005). Fundamentals of nursing (6th ed.). St. Louis: Mosby, pp. 270-271, 375, 391. 1533. {PLACE FIGURE HERE (Fig. 14)} Wilson, S. & Giddens, J. (2005). Health assessment for nursing practice (3rd ed.). St. Louis: Mosby, p. 348. A nurse is collecting respiratory data from a client and is auscultating for normal breath sounds. The nurse would expect to hear bronchial breath sounds in which anatomical area? Answer: 2 Rationale: There are three types of normal breath sounds in the adult and older child. These include bronchial (sometimes called tracheal or tubular), bronchovesicular, and vesicular. Bronchial breath sounds are heard over the trachea and larynx. Bronchovesicular breath sounds are heard over the major bronchi. Vesicular breath sounds are heard over peripheral lung fields where the air enters the alveoli. Test-Taking Strategy: Use knowledge regarding respiratory physical assessment techniques. Recalling that bronchial breath sounds are also known as tracheal or tubular will assist in answering the question. Review these types of breath sounds if you had difficulty with this question. Level of Cognitive Ability: Analysis

PN~CD~Questions~1501-1600-

20

Client Needs: Health Promotion and Maintenance Integrated Process: Nursing Process/Data Collection Content Area: Adult Health/Respiratory Reference: Wilson, S., & Giddens, J. (2005). Health assessment for nursing practice (3rd ed.). St. Louis: Mosby, p. 348. 1534. A nursing instructor asks a nursing student to describe the standards of care formulated by the American Nurses Association. Which statement by the student indicates an inaccurate description of these statements? 1. “They are authoritative statements that describe a common or acceptable level of client care or performance.” 2. “They have some similarity to policies and procedures.” 3. “They are specific guidelines.” 4. “They define professional practice.” Answer: 3 Rationale: Standards of care are authoritative statements that describe a common or acceptable level of client care or performance that have some similarity to policies and procedures. Thus standards of care define professional practice. The American Nurses Association has formulated general standards and guidelines for nursing practice. They are broad and general in nature and apply across the nation. Test-Taking Strategy: Use the process of elimination noting the key word inaccurate in the stem of the question. These words indicate a false-response question and that you need to select the incorrect student statement. Noting the issue, standards of care formulated by the American Nurses Association, will direct you to option 3. Review these standards of care if you had difficulty with this question. Level of Cognitive Ability: Comprehension Client Needs: Safe, Effective Care Environment Integrated Process: Teaching/Learning Content Area: Leadership/Management Reference: Potter, P., & Perry, A. (2005). Fundamentals of nursing (6th ed.). St. Louis: Mosby, pp. 365, 407-408. 1535. A nurse discovers that one of her assigned clients is bleeding excessively from an abdominal incision. The nurse gives specific orders to a nursing assistant to attend to the other clients and tells another nurse to call the physician immediately. In this situation, the nurse is implementing what leadership style? 1. Democratic 2. Situational 3. Autocratic 4. Laissez-faire Answer: 3 Rationale: Autocratic leadership, also called directive leadership, involves the leader assuming complete control over the decisions and activities of the group. In this situation, the nurse assumed the autocratic style of leadership so that all necessary tasks would be accomplished immediately. Democratic leadership, also called participative leadership, is characterized by a sense of equality among the leader and other

PN~CD~Questions~1501-1600-

21

participants. Situational leadership is a comprehensive approach that incorporates the leader’s style, the maturity of the work group, and the situation at hand. Laissez-faire is a permissive style of leadership in which the leader gives up control and delegates all decision making to the work group. Test-Taking Strategy: Use the process of elimination and focus on the data in the question. Noting that the nurse assumed complete control over the decisions and activities of the group will direct you to option 3. Review the various types of leadership styles if you had difficulty with this question. Level of Cognitive Ability: Application Client Needs: Safe, Effective Care Environment Integrated Process: Nursing Process/Implementation Content Area: Leadership/Management Reference: Zerwekh, J., & Claborn, J. (2003). Nursing today: Transitions and trends (4th ed.). Philadelphia: W.B. Saunders, pp. 103-104. 1536. A nurse is reviewing the record of a client with cervical cancer. Which of the following would the nurse expect to note in the client’s record related to a risk factor associated with this type of cancer? 1. Single female, no children 2. Intercourse with a single sex partner 3. Intercourse with circumcised males 4. History of human papilloma virus Answer: 4 Rationale: Risk factors associated with cervical cancer include intercourse with uncircumcised males, early frequent intercourse with multiple sexual partners, multiparity, chronic cervicitis, and history of genital herpes or human papilloma virus infection. Cervical cancer is also higher in the black race. Test-Taking Strategy: Knowledge regarding the risk factors associated with cervical cancer is required to answer this question. Using the process of elimination and this knowledge and careful reading of the options will direct you to option 4. If you had difficulty with this question, review the risks of cervical cancer. Level of Cognitive Ability: Analysis Client Needs: Physiological Integrity Integrated Process: Nursing Process/Data Collection Content Area: Adult Health/Oncology Reference: Linton, A., & Maebius, N. (2003). Introduction to medical-surgical nursing (3rd ed.). Philadelphia: W.B. Saunders, p. 959. 1537. A client reports to the health care clinic for an eye examination, and a diagnosis of primary open-angle glaucoma is suspected. Which nursing assessment question will elicit information regarding the initial clinical manifestations associated with this disorder? 1. “Do you have any pain in your eyes?” 2. “Have you had difficulty with peripheral vision?” 3. “Do bright lights cause a glaring?” 4. “Is your central vision blurred?”

PN~CD~Questions~1501-1600-

22

Answer: 2 Rationale: Because glaucoma is usually symptom free, the client may first note changes in peripheral visual acuity. If pain occurs with glaucoma, it is usually late in the course of structural changes with an intraocular pressure of 40 to 50 mm Hg or higher. Most severe pain is characteristic of absolute glaucoma (total vision loss). Glare from bright lights is a complaint of a client with a cataract. Blurred central vision occurs with macular degeneration. Test-Taking Strategy: Knowledge regarding the clinical manifestations associated with glaucoma is required to answer this question. Remember that because glaucoma is usually symptom free, the client may first note changes in peripheral visual acuity. If you are unfamiliar with these signs, review the clinical manifestations of this disorder. Level of Cognitive Ability: Analysis Client Needs: Physiological Integrity Integrated Process: Nursing Process/Data Collection Content Area: Adult Health/Eye Reference: Linton, A., & Maebius, N. (2003). Introduction to medical-surgical nursing (3rd ed.). Philadelphia: W.B. Saunders, pp. 1066-1067. 1538. A nurse is preparing to reinforce instructions to a client with glaucoma regarding the prescribed treatment measures for the disorder. The nurse prepares the instructions based on the primary objective of: 1. Maintaining intraocular pressure at a reduced level 2. Producing mydriasis in the eyes 3. Increasing the formation of aqueous humor 4. Promoting dilation of the pupil of the eyes Answer: 1 Rationale: The principle of treatment of the client with glaucoma is to maintain intraocular pressure at a reduced level to prevent further damage to intraocular structures. Medications are used to create miosis (constriction of the pupil) and to reduce formation of the aqueous humor by the ciliary body. Test-Taking Strategy: Use the process of elimination. Eliminate options 2 and 4 first because they are similar. Next recalling that glaucoma is a condition that is characterized by increased intraocular pressure will assist in eliminating option 3. Review the goals of treatment for the client with glaucoma if you had difficulty with this question. Level of Cognitive Ability: Application Client Needs: Physiological Integrity Integrated Process: Nursing Process/Planning Content Area: Adult Health/Eye Reference: Linton, A., & Maebius, N. (2003). Introduction to medical-surgical nursing (3rd ed.). Philadelphia: W.B. Saunders, p. 1067. 1539. A nurse in the outpatient unit is preparing a client who is scheduled for a laser trabeculoplasty for the treatment of primary open-angle glaucoma. Which of the following instructions should the nurse provide to the client? 1. “The procedure takes about 2 hours.” 2. “Activities can be resumed following the procedure immediately.”

PN~CD~Questions~1501-1600-

23

3. “You may return to work 1 to 2 days following the procedure.” 4. “Your vision loss will be restored following the procedure.” Answer: 3 Rationale: Laser trabeculoplasty is performed in the outpatient setting and requires about 30 minutes. The client will experience little discomfort and may resume all normal activities including returning to work within 1 to 2 days. The treatment prevents further visual loss, but the lost vision cannot be restored. Test-Taking Strategy: Knowledge regarding this procedure is required to answer this question. Option 4 can be eliminated first knowing that vision that is lost cannot be restored. Next eliminate option 2 because of the word “immediately.” From the remaining options, it is necessary to know that this procedure takes about 30 minutes. Review this procedure if you are unfamiliar with it. Level of Cognitive Ability: Application Client Needs: Physiological Integrity Integrated Process: Nursing Process/Implementation Content Area: Adult Health/Eye References: Black, J., & Hawks, J. (2005). Medical-surgical nursing: Clinical management for positive outcomes (7th ed.). Philadelphia: W.B. Saunders, p. 1948. Lewis, S., Heitkemper, M., & Dirksen, S. (2004). Medical-surgical nursing: Assessment and management of clinical problems (6th ed.). St. Louis: Mosby, pp. 457-458. 1540. A client with multiple sclerosis is receiving baclofen (Lioresal). The nurse monitoring this client should look for which of the following to indicate a primary therapeutic response from the medication? 1. Decreased muscle spasms 2. Increased range of motion of all extremities 3. Increased muscle tone and strength 4. Decreased nausea Answer: 1 Rationale: Baclofen is a skeletal muscle relaxant and acts at the spinal cord level to decrease the frequency and amplitude of muscle spasms in clients with spinal cord injuries or diseases and multiple sclerosis. Options 2 and 3 are not directly related to the effects of this medication. Option 4 is incorrect. Test-Taking Strategy: Note that the key word in the stem of the question is primary. Knowledge that this medication is a skeletal muscle relaxant guides you to the correct option. Review this medication if you had difficulty with this question. Level of Cognitive Ability: Analysis Client Needs: Physiological Integrity Integrated Process: Nursing Process/Evaluation Content Area: Pharmacology Reference: Hodgson, B., & Kizior, R. (2005). Saunders nursing drug handbook 2005. Philadelphia: W.B. Saunders, p. 107. 1541. A nurse is collecting data regarding the motor development of a 24-month-old child. Based on the age of the child, the nurse expects to note which highest level of

PN~CD~Questions~1501-1600-

24

developmental milestones? 1. The child builds a tower of two blocks 2. The child opens a door knob 3. The child snaps large snaps 4. The child puts on simple clothes independently Answer: 2 Rationale: A 24-month-old would be able to open a door knob. At age 15 months, the nurse would expect that the child could build a tower of two blocks. At age 30 months, the child would be able to snap large snaps and put on simple clothes independently. Test-Taking Strategy: Use the process of elimination. Note the age of the child. Visualize each of the fine motor skills to assist in selecting the correct option. Review the developmental milestones if you had difficulty with this question. Level of Cognitive Ability: Comprehension Client Needs: Health Promotion and Maintenance Integrated Process: Nursing Process/Data Collection Content Area: Child Health Reference: Leifer, G. (2003). Introduction to maternity & pediatric nursing (4th ed.). Philadelphia: W.B. Saunders, p. 407. 1542. A severe, pounding headache develops in a client with a spinal cord injury. The client is diaphoretic, hypertensive and bradycardic, and complains of nausea and nasal congestion. The nurse determines that the client is experiencing autonomic hyperreflexia (autonomic dysreflexia). Place in order of priority the actions that the nurse should take. Notifies the physician Documents the findings Places the client in sitting position Checks the bladder for distention Answer: Places the client in sitting position Checks the bladder for distention Notifies the physician Documents the findings Rationale: Autonomic hyperreflexia is an acute emergency that occurs as a result of exaggerated autonomic responses to stimuli that are innocuous in normal individuals. It occurs only after spinal shock has resolved. A number of stimuli may trigger this response, including a distended bladder (the most common cause); distention or contraction of the visceral organs, especially the bowel (from constipation or impaction); or stimulation of the skin. When autonomic hyperreflexia occurs, the client is immediately placed in a sitting position to lower the blood pressure. The nurse would then perform a rapid assessment to identify and alleviate the cause. The client’s bladder is emptied immediately via a urinary catheter, the rectum is checked for the presence of a fecal mass, and the skin is examined for areas of pressure, irritation, or broken skin. The physician is notified, and the nurse documents the occurrence and the actions taken. Test-Taking Strategy: Focus on the data in the question and note that the client is hypertensive. Placing the client in a sitting position will lower the blood pressure, and

PN~CD~Questions~1501-1600-

25

this action would be done first. Remember that the nurse needs to quickly eliminate the cause next. The physician is then notified, and the nurse documents the occurrence and the actions taken. Review care to the client experiencing autonomic hyperreflexia if you had difficulty with this question. Level of Cognitive Ability: Application Client Needs: Safe, Effective Care Environment Integrated Process: Nursing Process/Implementation Content Area: Delegating/Prioritizing Reference: Black, J., & Hawks, J. (2005). Medical-surgical nursing: Clinical management for positive outcomes (7th ed.). Philadelphia: W.B. Saunders, pp. 2215, 2229. 1543. A nurse is caring for a client who had a below-the-knee amputation of the right leg with a cast on the residual limb. The client calls the nurse and reports that the cast fell off. The nurse immediately: 1. Replaces the cast with a new one 2. Contacts the surgeon 3. Documents the findings 4. Wraps the residual limb with an elastic compression bandage Answer: 4 Rationale: If a cast or elastic dressing inadvertently comes off of the residual limb of a client with an amputation, the nurse immediately wraps the residual limb with an elastic compression bandage. If this is not done immediately, excessive edema will develop in a short period of time. The nurse does not replace a cast. The nurse would notify the surgeon if a cast came off so that another cast can be applied. The nurse would document the occurrence and the actions taken. Test-Taking Strategy: Use the process of elimination noting the key word immediately. Eliminate option 1 because replacing a cast is not within the realm of a nurse’s responsibilities. Recalling that the purpose of the cast is to shape the residual limb for prosthesis fitting will direct you to option 4 from the remaining options. Review care to the client following amputation if you had difficulty with this question. Level of Cognitive Ability: Application Client Needs: Safe, Effective Care Environment Integrated Process: Nursing Process/Implementation Content Area: Adult Health/Musculoskeletal Reference: Linton, A., & Maebius, N. (2003). Introduction to medical-surgical nursing (3rd ed.). Philadelphia: W.B. Saunders, p. 849. 1544. A nurse assists in administering first aid to a client who has been bitten by a snake on the right leg. The nurse understands that the initial action is to: 1. Apply ice to the site of the bite 2. Apply a tourniquet 3. Ensure that the victim is lying down and remove restrictive items 4. Elevate the leg above the level of the heart Answer: 3 Rationale: Initial first aid at the site of a snake bite includes having the victim lie down,

PN~CD~Questions~1501-1600-

26

removing constrictive items, such as clothing or rings, providing warmth, cleansing the wound, covering the wound with a light sterile dressing, and immobilizing the injured body part below the level of the heart. Ice or a tourniquet is not applied during the acute stage. Additionally, corticosteroids are contraindicated in the first 6 to 8 hours after the bite because they may depress antibody production and hinder the action of antivenin (antitoxin manufactured from the snake venom). Test-Taking Strategy: Use the process of elimination and prioritization skills noting the key word initial. Use of the principles related to the effects of ice, limb constriction, and gravity will direct you to option 3. Review initial care to the client who sustained a snake bite if you had difficulty with this question. Level of Cognitive Ability: Application Client Needs: Physiological Integrity Integrated Process: Nursing Process/Implementation Content Area: Fundamental Skills References: Black, J., & Hawks, J. (2005). Medical-surgical nursing: Clinical management for positive outcomes (7th ed.). Philadelphia: W.B. Saunders, p. 2505. Linton, A., & Maebius, N. (2003). Introduction to medical-surgical nursing (3rd ed.). Philadelphia: W.B. Saunders, p. 203. 1545. A nurse provides a list of instructions to a client with glaucoma regarding measures that will prevent an increase in intraocular pressure in the eyes. Which statement by the client indicates a need for further instructions? 1. “I can move objects weighing 20 lb or more by pushing the object on the floor using the feet.” 2. “I can tie my shoelaces by bending over slowly.” 3. “I need to consume a diet high in fiber and bulk.” 4. “I need to maintain an intake of six to eight glasses of water a day.” Answer: 2 Rationale: Activities, such as bending over or straining at stool, will increase intraocular pressure. The client needs to be instructed to maintain a diet high in bulk and fiber and to consume a high intake of liquids, unless contraindicated, to prevent constipation and straining at stools. The client should tie shoelaces by bending the knee, raising the thigh, and bringing the foot within hand reach. Objects weighing 20 lb or more can be moved by pushing the object on the floor using the feet or by the use of a mechanical dolly. Test-Taking Strategy: Note the key words need for further instructions. These words indicate a false-response question and that you need to select the incorrect client statement. Using the principles related to measures that will prevent an increase in intraocular pressure will direct you to option 2. Review these measures if you had difficulty with this question. Level of Cognitive Ability: Comprehension Client Needs: Health Promotion and Maintenance Integrated Process: Teaching/Learning Content Area: Adult Health/Eye Reference: Linton, A., & Maebius, N. (2003). Introduction to medical-surgical nursing (3rd ed.). Philadelphia: W.B. Saunders, p. 1070.

PN~CD~Questions~1501-1600-

27

1546. A clinic nurse is reviewing the record of a client recently diagnosed with a cataract. Which clinical manifestation associated with this disorder would the nurse expect to be documented in the client’s record? 1. Constant, dull, and achy pain in the eyes 2. Loss of central vision only 3. Painless progressive loss of vision 4. Color blindness Answer: 3 Rationale: A cataract is any opacity of the crystalline lens of the eye. The classic symptom of cataract is painless progressive loss of vision in one or both eyes. Some individuals also complain of glare from bright lights. Occasionally, pain can result when the lens becomes swollen and blocks the normal flow of aqueous fluid causing an increased intraocular pressure. Color blindness is not an associated symptom. Test-Taking Strategy: Knowledge that a cataract is an opacity of the lens of the eye will assist in answering the question. Eliminate option 2 first because of the word “only.” Next eliminate option 4 knowing that color blindness is not an associated manifestation. Knowing that this disorder is painless will direct you to option 3. Review the manifestations associated with cataract if you had difficulty with this question. Level of Cognitive Ability: Comprehension Client Needs: Physiological Integrity Integrated Process: Nursing Process/Data Collection Content Area: Adult Health/Eye Reference: Christensen, B., & Kockrow, E. (2003). Adult health nursing (4th ed.). St. Louis: Mosby, p. 570. 1547. Prescriptive glasses are prescribed for a client with bilateral aphakia, and the nurse provides instructions to the client regarding the use of the glasses. Which statement by the client indicates the need for further instructions? 1. “The prescriptive glasses will correct my visual field of sight.” 2. “The prescriptive glasses will magnify my central vision by 30%.” 3. “Objects that I look at may be distorted.” 4. “It may be difficult to judge distances when I drive a car.” Answer: 1 Rationale: Aphakia (absence of the lens of the eye) can be corrected by prescriptive glasses, contact lens, or intraocular lens. Only central vision is corrected with these prescriptive glasses, and the peripheral vision is distorted. There is approximately 30% magnification of central vision with prescriptive glasses. This requires adjustment to daily activities and safety precautions. Because of the magnification, objects viewed centrally appear distorted, and it is difficult to judge distances, such as when driving a car or sitting in a chair. Test-Taking Strategy: Note the key words need for further instructions. These words indicate a false-response question and that you need to select the incorrect client statement. Think about the use of glasses and the visual field as you answer the question. This thinking process will direct you to option 1. If you had difficulty with this question, review client teaching points related to the use of these glasses.

PN~CD~Questions~1501-1600-

28

Level of Cognitive Ability: Comprehension Client Needs: Health Promotion and Maintenance Integrated Process: Teaching/Learning Content Area: Adult Health/Eye References: Ignatavicius, D., & Workman, M. (2006). Medical-surgical nursing: Critical thinking for collaborative care (5th ed.). Philadelphia: W.B. Saunders, p. 1094. Lewis, S., Heitkemper, M., & Dirksen, S. (2004). Medical-surgical nursing: Assessment and management of clinical problems (6th ed.). St. Louis: Mosby, pp. 1033-1034. 1548. A client is brought to the ambulatory care department by the spouse one day following a cataract extraction procedure. A diagnosis of hyphema is made, which occurred as a result of the surgical procedure. The nurse reinforces instructions to the client and spouse regarding the treatment for the complication and makes which statement to them? 1. “Resume normal activities because the hyphema will resolve on its own.” 2. “Ambulate as necessary.” 3. “Return to the outpatient department for removal of the intraocular lens implant.” 4. “Maintain bed rest and patching of both eyes.” Answer: 4 Rationale: Hyphema is bleeding into the anterior chamber of the eye that occurs postoperatively as a complication of cataract surgery. Treatment includes bed rest and bilateral eye patching for 2 to 5 days during which absorption occurs. The client should be instructed to monitor for signs of increased intraocular pressure, which commonly causes sudden ocular pain. Miotics and cycloplegics may be prescribed. Occasionally, irrigation of the anterior chamber may be done to remove the blood. Test-Taking Strategy: Use the process of elimination. Eliminate options 1 and 2 first because they are similar. Next eliminate option 3 because this is unnecessary, and there is no data in the question that indicates that an intraocular implant was performed. If you had difficulty with this question, review the treatment for a hyphema. Level of Cognitive Ability: Application Client Needs: Physiological Integrity Integrated Process: Nursing Process/Implementation Content Area: Adult Health/Eye References: Ignatavicius, D., & Workman, M. (2006). Medical-surgical nursing: Critical thinking for collaborative care (5th ed.). Philadelphia: W.B. Saunders, p. 1105. Lewis, S., Heitkemper, M., & Dirksen, S. (2004). Medical-surgical nursing: Assessment and management of clinical problems (6th ed.). St. Louis: Mosby, p. 422. 1549. A nurse receives a telephone call from a neighbor who states that her child was found sitting on the floor near the kitchen sink playing with several bottles of cleaning fluids. The bottles of cleaning fluid were opened and spilled on the child and the floor, and the mother suspects that the child may have drunk some of the cleaning fluid. The nurse tells the mother to immediately: 1. Call an ambulance to bring the child to the emergency room 2. Call the physician 3. Call the area poison control center

PN~CD~Questions~1501-1600-

29

4. Call the drug store to purchase syrup of ipecac for administration Answer: 3 Rationale: The area poison control center should be called if an unknown toxic agent has been ingested or if it is necessary to identify an antidote for a known toxic agent. Syrup of ipecac is used to induce vomiting in a conscious client, but vomiting is not induced after ingestion of caustic substances (acid or alkaline) or petroleum distillates. Additionally, it is unadvisable for parents to administer syrup of ipecac, and this action if warranted should be done under medical supervision. Calling an ambulance or calling the physician will delay necessary life-saving measures. Test-Taking Strategy: Use the process of elimination and focus on the data in the question. The specific poison ingested is unknown. Therefore the only option that will provide information about immediate care is option 3. Review immediate care to the client who ingested poison if you had difficulty with this question. Level of Cognitive Ability: Application Client Needs: Safe, Effective Care Environment Integrated Process: Nursing Process/Implementation Content Area: Fundamental Skills References: Linton, A., & Maebius, N. (2003). Introduction to medical-surgical nursing (3rd ed.). Philadelphia: W.B. Saunders, p. 200. Potter, P., & Perry, A. (2005). Fundamentals of nursing (6th ed.). St. Louis: Mosby, pp. 992-993. 1550. A nurse discusses emergency nursing measures that are implemented at the site of an injury with a nursing student. The nurse tells the student that in the event of carbon monoxide poisoning, the initial action is to: 1. Wrap the client in blankets 2. Keep the client as quiet as possible 3. Carry the client to fresh air 4. Initiate cardiopulmonary resuscitation (CPR) Answer: 3 Rationale: Whenever a victim inhales a poison, the victim is immediately carried to fresh air. Any tight clothing is then loosened, and CPR is initiated if necessary. Oxygen is administered as soon as possible. Chilling is prevented, and the victim is wrapped in blankets and kept as quiet as possible. Test-Taking Strategy: Use the process of elimination focusing on the data in the question. Noting that the question relates to an inhalation of a poison and addresses care at the site of an injury will direct you to option 3. Remember that CPR is initiated only if required. Review care to the client who inhaled a poison if you had difficulty with this question. Level of Cognitive Ability: Application Client Needs: Safe, Effective Care Environment Integrated Process: Teaching/Learning Content Area: Leadership/Management Reference: Black, J., & Hawks, J. (2005). Medical-surgical nursing: Clinical management for positive outcomes (7th ed.). Philadelphia: W.B. Saunders, pp. 1906-1907.

PN~CD~Questions~1501-1600-

30

1551. A client who sustained a severe burn injury is brought to the emergency room. The nurse prepares to implement which immediate action? 1. Assess neurological function 2. Administer 100% humidified oxygen 3. Cover the client with a warm blanket. 4. Insert an endotracheal tube Answer: 2 Rationale: When a victim who sustains a burn injury arrives at the emergency room, breathing is assessed, and a patent airway is established immediately. The client is also immediately given 100% humidified oxygen. If the victim has severe respiratory distress or airway edema, then an endotracheal tube is inserted for mechanical ventilation. Clean sheets are placed over the client, and the client’s body temperature is maintained. The client’s neurological status is assessed; however, this is not the immediate action. Test-Taking Strategy: Use the process of elimination and the ABCs—airway, breathing, and circulation—to answer the question. Focusing on the key words immediate action will direct you to option 2. Review care to a burn client if you had difficulty with this question. Level of Cognitive Ability: Application Client Needs: Safe, Effective Care Environment Integrated Process: Nursing Process/Implementation Content Area: Delegating/Prioritizing Reference: Linton, A., & Maebius, N. (2003). Introduction to medical-surgical nursing (3rd ed.). Philadelphia: W.B. Saunders, p. 1037. 1552. A nurse is assisting in caring for a victim of a burn injury during the emergent and/or resuscitative phase. On data collection of the client, the nurse notes that the urine output has decreased, and the blood pressure is dropping. The nurse would immediately: 1. Increase the intravenous (IV) flow rate 2. Cover the client with a warm blanket 3. Notify the registered nurse (RN) 4. Reassess the client in 30 minutes Answer: 3 Rationale: The nurse notifies the RN who will then notify the physician immediately if the burn client exhibits a decreased urine output; blood pressure; or an increased pulse rate. Because of the rapid fluid shifts that occur in burn shock, fluid deficit must be detected early so that distributive shock does not occur. A nurse does not increase an IV rate without a specific order to do so. Reassessing the client in 30 minutes will delay necessary interventions to prevent the development of distributive shock. A warm environment is maintained, but this is not the immediate action. Test-Taking Strategy: Use the process of elimination noting the key word immediately. Recalling the complications associated with a burn injury will direct you to option 3. Review these complications if you had difficulty with this question. Level of Cognitive Ability: Application Client Needs: Safe, Effective Care Environment Integrated Process: Nursing Process/Implementation Content Area: Adult Health/Integumentary

PN~CD~Questions~1501-1600-

31

Reference: Linton, A., & Maebius, N. (2003). Introduction to medical-surgical nursing (3rd ed.). Philadelphia: W.B. Saunders, p. 1037. 1553. A nurse receives a client in the surgical unit who was transferred from the postanesthesia care unit. The nurse assesses the client for which of the following first? 1. Active bowel sounds 2. Adequate urine output 3. Orientation to the surroundings 4. A patent airway Answer: 4 Rationale: After transfer from the postanesthesia care unit, the nurse performs an assessment on the client. The ABCs—airway, breathing, and circulation—must be assessed first. Urine output and orientation to the surroundings may also be assessed, but these are not the first actions. The client may not have active bowel sounds at this time because of the effects of anesthesia. Test-Taking Strategy: Use the process of elimination and note the key word first. Use of the ABCs—airway, breathing, and circulation—will direct you to option 4. Review the initial actions when caring for a client following surgery if you had difficulty with this question. Level of Cognitive Ability: Application Client Needs: Safe, Effective Care Environment Integrated Process: Nursing Process/Implementation Content Area: Delegating/Prioritizing Reference: Christensen, B., & Kockrow, E. (2003). Adult health nursing (4th ed.). St. Louis: Mosby, p. 41. 1554. A nurse is reinforcing preoperative instructions to a client scheduled for cataract surgery and prepares a written list of instructions for the client. Which statement by the client indicates a need for further instructions? 1. “I can drink any liquids that I want to on the morning of the surgery.” 2. “Medication may be given to me on the day of surgery to produce relaxation.” 3. “My eyelashes may be cut before surgery.” 4. “Eye medications will be placed in my eye before the surgery.” Answer: 1 Rationale: The client should be instructed that no oral intake is permitted for 6 to 12 hours before the surgical procedure. Local or general anesthetic will be administered, and the client may receive medication to produce relaxation. Eyelashes may be cut before surgery and will grow back, but will grow slowly. Eye medications, such as mydriatics, cycloplegics, or beta-blockers, may be administered before the surgical procedure. Test-Taking Strategy: Note the key words need for further instructions. These words indicate a false-response question and that you need to select the incorrect client statement. Knowledge that an anesthetic of some form is administered before this surgical procedure will assist in directing you to option 1. Review preoperative client instructions if you had difficulty with this question. Level of Cognitive Ability: Comprehension

PN~CD~Questions~1501-1600-

32

Client Needs: Physiological Integrity Integrated Process: Teaching/Learning Content Area: Adult Health/Eye References: Black, J., & Hawks, J. (2005). Medical-surgical nursing: Clinical management for positive outcomes (7th ed.). Philadelphia: W.B. Saunders, pp. 276-277. Lewis, S., Heitkemper, M., & Dirksen, S. (2004). Medical-surgical nursing: Assessment and management of clinical problems (6th ed.). St. Louis: Mosby, p. 450. Phipps, W., Monahan, F., Sands, J., Marek, J., & Neighbors, M. (2003). Medical-surgical nursing: Health and illness perspectives (7th ed.). St. Louis: Mosby, p. 1901. 1555. A nurse collects data from a client with macular degeneration of the eye. The nurse would expect the client to report which of the following symptoms? 1. Loss of peripheral vision 2. Blurred central vision 3. Increased clarity when looking at objects 4. Clear vision when reading Answer: 2 Rationale: The most common symptom of macular degeneration is blurred central vision that often occurs suddenly. Clients complain of difficulty with reading and seeing fine detail. Formation of a central scotoma (blind spot) occurs in some clients. Clients may complain of visual distortion usually described as a bending or irregularity of straight lines. Peripheral vision is spared, so although affected persons cannot see to read, drive, watch television clearly, or distinguish faces, they do have the ability to walk. Test-Taking Strategy: Focus on the client’s diagnosis. This will assist in eliminating options 3 and 4. From the remaining options, remember that the most common symptom of macular degeneration is blurred central vision. Review these symptoms if you had difficulty with this question. Level of Cognitive Ability: Comprehension Client Needs: Physiological Integrity Integrated Process: Nursing Process/Data Collection Content Area: Adult Health/Eye Reference: Linton, A., & Maebius, N. (2003). Introduction to medical-surgical nursing (3rd ed.). Philadelphia: W.B. Saunders, p. 1071. 1556. A nurse is collecting data from a client who is admitted to the hospital for diagnostic studies to rule out the presence of Hodgkin’s disease. Which question would the nurse ask the client to elicit information specifically related to this disease? 1. “Do you have any weakness?” 2. “Are you tiring easily?” 3. “Have you gained any weight?” 4. “Have you noticed any swollen lymph nodes?” Answer: 4 Rationale: Hodgkin’s disease is a chronic progressive neoplastic disorder of lymphoid tissue characterized by the painless enlargement of lymph nodes with progression to extralymphatic sites, such as the spleen and liver. Weight loss is most likely to be noted.

PN~CD~Questions~1501-1600-

33

Fatigue and weakness may occur, but is not significantly related to the disease. Test-Taking Strategy: Knowledge that Hodgkin’s disease affects the lymph nodes will easily direct you to option 4. Option 3 can be easily eliminated first because in such a disorder weight loss is most likely to occur. Options 1 and 2 are similar and rather vague symptoms that can occur in many disorders. Review the manifestations associated with Hodgkin’s disease if you had difficulty with this question. Level of Cognitive Ability: Application Client Needs: Physiological Integrity Integrated Process: Nursing Process/Data Collection Content Area: Adult Health/Oncology Reference: Christensen, B., & Kockrow, E. (2003). Adult health nursing (4th ed.). St. Louis: Mosby, p. 279. 1557. A nurse is collecting data from a client suspected of having ovarian cancer. Which question would the nurse ask the client to elicit information specifically related to this disorder? 1. “Are you having any excessive vaginal bleeding?” 2. “Does your abdomen feel as though it is swollen?” 3. “Have you been having diarrhea?” 4. “Have you had any abnormal vaginal bleeding?” Answer: 2 Rationale: Clinical manifestations of ovarian cancer include abdominal distention, urinary frequency and urgency, pleural effusion, malnutrition, pain from pressure caused by the growing tumor and the effects of urinary or bowel obstruction and constipation. Ascites with dyspnea and ultimately general severe pain will occur as the disease progresses. Abnormal bleeding, often resulting in hypermenorrhea, is associated with uterine cancer. Test-Taking Strategy: Use the process of elimination focusing on the issue, ovarian cancer. Eliminate options 1 and 4 first because they are similar. From the remaining options, consider the anatomical location of the diagnosis. This will assist in directing you to option 2. Review the manifestations associated with ovarian cancer if you had difficulty with this question. Level of Cognitive Ability: Application Client Needs: Physiological Integrity Integrated Process: Nursing Process/Data Collection Content Area: Adult Health/Oncology Reference: Linton, A., & Maebius, N. (2003). Introduction to medical-surgical nursing (3rd ed.). Philadelphia: W.B. Saunders, p. 960. 1558. A nurse notes that atrial fibrillation suddenly develops in a client who is attached to a cardiac monitor. The nurse notifies the registered nurse immediately and prepares the client for which initial intervention? 1. Administration of digoxin (Lanoxin) 2. Chemical cardioversion with quinidine (Cardioquin) 3. Electrical cardioversion 4. Heparin infusion therapy

PN~CD~Questions~1501-1600-

34

Answer: 1 Rationale: The initial treatment goal when atrial fibrillation suddenly occurs is to control the rate of impulses with the administration of medications, such as diltiazem (Cardizem), verapamil (Calan), beta-blockers, or digoxin. Chemical cardioversion, usually after a period of anticoagulation therapy, can then be attempted with procainamide (Pronestyl) or quinidine. Electrical cardioversion is the third therapeutic option. Test-Taking Strategy: Knowledge regarding the treatment for atrial fibrillation is required to answer this question. Noting the key words initial intervention will assist in eliminating options 3 and 4 because these are invasive treatments. From the remaining options, it is necessary to know that the initial treatment is the administration of medications, such as diltiazem, verapamil, beta-blockers, or digoxin. Review care to the client with atrial fibrillation if you had difficulty with this question. Level of Cognitive Ability: Analysis Client Needs: Physiological Integrity Integrated Process: Nursing Process/Planning Content Area: Adult Health/Cardiovascular Reference: Christensen, B., & Kockrow, E. (2003). Adult health nursing (4th ed.). St. Louis: Mosby, p. 298. 1559. A client who experienced ventricular fibrillation has just been defibrillated. Following the defibrillation, the nurse immediately: 1. Resumes cardiopulmonary resuscitation (CPR) 2. Checks the cardiac rhythm and pulse 3. Checks the client’s neurological status 4. Increases the intravenous flow rate Answer: 2 Rationale: Following defibrillation, the nurse immediately checks the client’s cardiac rhythm and pulse. These assessments will provide the information necessary to determine if the defibrillation was successful. If the first countershock is unsuccessful, immediate defibrillation is performed at a higher energy level. Defibrillation may be applied up to three times if needed for persistent ventricular fibrillation or pulseless ventricular fibrillation. CPR should be continued if the three defibrillations are unsuccessful. Options 3 and 4 are not immediate actions following defibrillation. Test-Taking Strategy: Use the process of elimination focusing on the issue, following the defibrillation. Recalling the purpose of defibrillation will direct you to option 2. Review the nursing actions related to defibrillation if you had difficulty with this question. Level of Cognitive Ability: Application Client Needs: Physiological Integrity Integrated Process: Nursing Process/Implementation Content Area: Adult Health/Cardiovascular Reference: Black, J., & Hawks, J. (2005). Medical-surgical nursing: Clinical management for positive outcomes (7th ed.). Philadelphia: W.B. Saunders, p. 1689. 1560. A nurse notes that the site of a client’s peripheral intravenous (IV) catheter is reddened, warm, painful, and slightly edematous proximal to the insertion point of the IV catheter. Based on these findings, the initial nursing action is to:

PN~CD~Questions~1501-1600-

35

1. Remove the IV 2. Slow the rate of infusion 3. Check for loose catheter connections 4. Notify the physician Answer: 1 Rationale: Phlebitis at an IV site can be determined by client discomfort at the site and by redness, warmth, and swelling proximal to the catheter. The line should be removed, and a new line should be inserted at a different site. Options 2 and 3 are incorrect. The physician would be notified if phlebitis occurred, but this is not the initial action. Test-Taking Strategy: Use the process of elimination and prioritization skills focusing on the data in the question. Eliminate options 2 and 3 because they are similar in that they indicate continuing with the IV therapy. Although the physician would be notified of this occurrence, the key word initial will direct you to option 1. Review these signs and the initial nursing actions if you had difficulty with this question. Level of Cognitive Ability: Application Client Needs: Physiological Integrity Integrated Process: Nursing Process/Implementation Content Area: Fundamental Skills Reference: Potter, P., & Perry, A. (2005). Fundamentals of nursing (6th ed.). St. Louis: Mosby, p. 1189. 1561. A nurse hangs a 1000-mL bag of intravenous (IV) fluid on an assigned client. Forty-five minutes later, the nurse notes that the client is complaining of a pounding headache, is dyspneic, apprehensive, and has an increased pulse rate. The IV bag has 500 mL remaining. The nurse should take which of the following actions first? 1. Sit the client up in bed 2. Slow the rate of infusion 3. Shut off the IV infusion 4. Remove the IV Answer: 3 Rationale: The client’s symptoms are compatible with speed shock. This may be verified by noting that 500 mL has infused in the course of 45 minutes. The first action of the nurse is to shut off the IV infusion. Other actions may follow in rapid sequence. The nurse may elevate the head of bed to aid the client’s breathing. The physician is also notified immediately. Slowing the infusion rate is inappropriate because the client will continue receiving fluid. The IV does not need to be removed. It may continue to be needed to manage the complication. Test-Taking Strategy: Use the process of elimination and prioritization skills focusing on the data in the question. Note the question contains the key word first. Also, recognizing the signs of speed shock and recalling the appropriate interventions will direct you to option 3. Review the initial nursing actions related to this complication if you had difficulty with this question. Level of Cognitive Ability: Application Client Needs: Physiological Integrity Integrated Process: Nursing Process/Implementation Content Area: Fundamental Skills

PN~CD~Questions~1501-1600-

36

Reference: Potter, P., & Perry, A. (2005). Fundamentals of nursing (6th ed.). St. Louis: Mosby, pp. 1173, 1179, 1189. 1562. A nurse is preparing to instruct a pregnant client about nutrition. The nurse plans to include which of the following in this client’s teaching plan? 1. The nutritional status of the mother significantly influences fetal growth and development 2. All mothers are at high risk for nutritional deficiencies 3. Calcium is not important until the third trimester 4. Iron supplements are not necessary unless the mother has iron deficiency anemia Answer: 1 Rationale: Poor nutrition during pregnancy can negatively influence fetal growth and development. Although pregnancy poses some nutritional risk for the mother, not all clients are at high risk. Calcium is critical during the third trimester, but must be increased from the onset of pregnancy. Intake of dietary iron is usually insufficient for the majority of pregnant women, and iron supplements are routinely prescribed. Test-Taking Strategy: Use the process of elimination. Eliminate option 2 because of the absolute word “all.” Also, note the absolute word “not” in options 3 and 4. Option 1 is a general statement true for any stage of pregnancy and is the most global (umbrella) option. Review the principles related to nutrition and pregnancy if you had difficulty with this question. Level of Cognitive Ability: Application Client Needs: Health Promotion and Maintenance Integrated Process: Teaching/Learning Content Area: Maternity/Antepartum Reference: Leifer, G. (2005). Maternity nursing (9th ed.). Philadelphia: W.B. Saunders, p. 59. 1563. A nurse is reinforcing instructions to a client scheduled for conization in 1 week for the treatment of microinvasive cervical cancer. The procedure has been explained by the physician, and the nurse is reviewing the complications associated with the procedure. The nurse determines that the client needs further instructions if the client states that which of the following is a complication of this procedure? 1. Ovarian perforation 2. Infertility 3. Infection 4. Incompetent cervix Answer: 1 Rationale: Conization is generally not performed on women who desire to bear children because it can lead to incompetence of the cervix or infertility. Complications of the procedure include hemorrhage, infection, and less frequently cervical stenosis. Test-Taking Strategy: Note the key words needs further instructions. These words indicate a false-response question and that you need to select the incorrect client statement. Also note the words “cervical cancer” in the question. Use the process of elimination and select option 1 because this option addresses an “ovarian” condition not a cervical one. Review the complications associated with this procedure if you had

PN~CD~Questions~1501-1600-

37

difficulty with this question. Level of Cognitive Ability: Analysis Client Needs: Physiological Integrity Integrated Process: Teaching/Learning Content Area: Adult Health/Oncology References: Black, J., & Hawks, J. (2005). Medical-surgical nursing: Clinical management for positive outcomes (7th ed.). Philadelphia: W.B. Saunders, p. 1074. Ignatavicius, D., & Workman, M. (2006). Medical-surgical nursing: Critical thinking for collaborative care (5th ed.). Philadelphia: W.B. Saunders, p. 1847. Lewis, S., Heitkemper, M., & Dirksen, S. (2004). Medical-surgical nursing: Assessment and management of clinical problems (6th ed.). St. Louis: Mosby, p. 1357. 1564. A nurse is reviewing the laboratory results of a client with bladder cancer and bone metastases and notes that the calcium level is 15 mg/dl. The nurse should take which appropriate action? 1. Notify the registered nurse (RN) 2. Document the findings 3. Increase calcium-containing foods in the diet 4. Ask the unit secretary to file the report in the client’s record Answer: 1 Rationale: Hypercalcemia is a serum calcium ion level greater than 11 mg/dl or 5.5 mEq/L. It most often occurs in clients who have bone metastases and is a late manifestation of extensive malignancy. The presence of cancer in the bone causes the bone to release calcium into the blood stream. Hypercalcemia is an oncological emergency. The nurse notifies the RN, who will then contact the physician. Test-Taking Strategy: Use the process of elimination and knowledge regarding the normal calcium level to answer the question. Knowing that the calcium level identified in the question indicates an elevated one will assist in eliminating option 3. Eliminate options 2 and 4 because they are similar actions. Review the normal calcium level and oncological emergencies if you had difficulty with this question. Level of Cognitive Ability: Application Client Needs: Physiological Integrity Integrated Process: Nursing Process/Implementation Content Area: Adult Health/Oncology References: Black, J., & Hawks, J. (2005). Medical-surgical nursing: Clinical management for positive outcomes (7th ed.). Philadelphia: W.B. Saunders, p. 387. Chernecky, C., & Berger, B. (2004). Laboratory tests and diagnostic procedures (4th ed.). Philadelphia: W.B. Saunders, p. 311. 1565. A nurse is reinforcing instructions to a client with a hordeolum regarding the treatment plan. Which of the following will the nurse include in the instructions to the client? 1. Apply cool compress to the eye twice daily 2. Apply warm compress for 15 minutes four times daily

PN~CD~Questions~1501-1600-

38

3. Press on the hordeolum after the warm compress to induce rupture 4. Avoid the use of antibiotic ointment Answer: 2 Rationale: A hordeolum is commonly known as a sty. Therapeutic management includes the application of warm compress for 15 minutes four times daily and instillation of an ophthalmic antibiotic ointment to combat the infectious organism and prevent the spread of infection to surrounding eyelid glands. The warm compress promotes comfort and aids in bringing purulent contents to a head, causing rupture with drainage. If a sty does not rupture spontaneously, it can be incised with a small sterile instrument by the physician. The client should be told not to press on or squeeze the sty to induce rupture because such pressure could force infectious material into the venous system and transmit infection to the brain. Test-Taking Strategy: Use the process of elimination. Read each option carefully thinking about the effects that each action would have on this type of eye disorder. This should direct you to option 2. If you are unfamiliar with the care of a hordeolum, review this content. Level of Cognitive Ability: Application Client Needs: Health Promotion and Maintenance Integrated Process: Teaching/Learning Content Area: Adult Health/Eye Reference: Christensen, B., & Kockrow, E. (2003). Adult health nursing (4th ed.). St. Louis: Mosby, p. 567. 1566. A client is seen in the health care clinic, and a diagnosis of conjunctivitis is made. The nurse reinforces discharge instructions to the client regarding care of the disorder while at home. Which statement by the client indicates a need for further instructions? 1. “I do not need to be concerned about spreading this infection to others in my family.” 2. “I should apply warm compress before instilling antibiotic drops if purulent discharge is present in my eye.” 3. “I should perform a saline eye irrigation before instilling the antibiotic drops into my eye if purulent discharge is present.” 4. “I can use an ophthalmic analgesic ointment at night if I have eye discomfort.” Answer: 1 Rationale: Conjunctivitis is highly contagious. Antibiotic drops are usually administered four times a day. When purulent discharge is present, saline eye irrigations or eye applications of warm compresses may be necessary before instilling the medication. Ophthalmic analgesic ointment or drops may be instilled especially at bedtime because discomfort becomes more noticeable when the eyelids are closed. Test-Taking Strategy: Note the key words need for further instructions. These words indicate a false-response question and that you need to select the incorrect client statement. Simply knowing that this disorder is considered highly contagious will direct to option 1. If you had difficulty with this question, review management of the client with this disorder. Level of Cognitive Ability: Comprehension Client Needs: Health Promotion and Maintenance Integrated Process: Teaching/Learning

PN~CD~Questions~1501-1600-

39

Content Area: Adult Health/Eye Reference: Christensen, B., & Kockrow, E. (2003). Adult health nursing (4th ed.). St. Louis: Mosby, p. 568. 1567. A nurse in the ambulatory care unit is caring for a client following cataract extraction. The client suddenly complains of nausea and severe eye pain in the surgical eye. The nurse should take which of the following actions immediately? 1. Apply ice to the operative eye 2. Turn the client onto the operative side 3. Notify the registered nurse (RN) 4. Administer the ordered pain medication and antiemetic Answer: 3 Rationale: Severe pain or pain accompanied by nausea is an indicator of increased intraocular pressure and should be reported to the RN who will notify the physician immediately. The other options are incorrect nursing actions. Ice is not applied to the surgical site unless prescribed. The client is not positioned on the operative side because of the risk of increasing intraocular edema from swelling. Although pain medication and an antiemetic may be prescribed, the client’s symptoms indicate a serious complication requiring physician notification. Test-Taking Strategy: Use the process of elimination and prioritization skills. Note the key words severe and immediately. Recalling that severe pain or pain accompanied by nausea is an indicator of increased intraocular pressure will direct you to option 3. If you had difficulty with this question, review the postoperative complications of cataract surgery. Level of Cognitive Ability: Application Client Needs: Physiological Integrity Integrated Process: Nursing Process/Implementation Content Area: Adult Health/Eye Reference: Christensen, B., & Kockrow, E. (2003). Adult health nursing (4th ed.). St. Louis: Mosby, p. 572. 1568. A client arrives at the emergency room after experiencing a traumatic blow to the eye, and a hyphema is diagnosed. The nurse immediately positions the client: 1. Prone 2. Supine 3. In Trendelenburg’s position 4. In semi-Fowler position Answer: 4 Rationale: A hyphema is the presence of blood in the anterior chamber of the eye. It is caused by an event that ruptures blood vessels in the eye, such a as penetrating injury from a BB pellet, or indirectly from a blow to the forehead. The client is treated with bed rest in a semi-Fowler position to assist gravity in keeping the hyphema away from the optical center of the cornea. The positions identified in options 1, 2, and 3 will be harmful to the client. Test-Taking Strategy: Use the process of elimination noting the key word immediately. Use knowledge of this condition and note that options 1, 2, and 3 are similar in that the

PN~CD~Questions~1501-1600-

40

client’s head will be flat. Review the immediate care measures for a client with hyphema if you had difficulty with this question. Level of Cognitive Ability: Application Client Needs: Physiological Integrity Integrated Process: Nursing Process/Implementation Content Area: Adult Health/Eye Reference: Ignatavicius, D., & Workman, M. (2006). Medical surgical nursing: Critical thinking for collaborative care (5th ed.). Philadelphia: W.B. Saunders, p. 1105. 1569. A client who was hit in the eye with a baseball bat sustains a contusion of the eyeball. The emergency room nurse implements which immediate action? 1. Applies ice to the affected eye 2. Places an eye patch on the affected eye 3. Applies a warm saline compress 4. Irrigates the eye with cool water Answer: 1 Rationale: Treatment for a contusion ideally begins at the time of injury and includes applying ice to the site. The client should also receive a thorough eye examination to rule out the presence of other eye injuries. An eye patch will not assist in treating this type of injury. Irrigating the eye with cool water may be implemented for injuries that involve a splash of an irritant into the eye. Test-Taking Strategy: Use the process of elimination and prioritization skills to answer the question. Focusing on the type of injury sustained will direct you to option 1. Also, use of the principles related to heat and cold therapy will assist in selecting the correct option. Review initial treatment following an eye contusion if you had difficulty with this question. Level of Cognitive Ability: Application Client Needs: Physiological Integrity Integrated Process: Nursing Process/Implementation Content Area: Adult Health/Eye Reference: Ignatavicius, D., & Workman, M. (2006). Medical-surgical nursing: Critical thinking for collaborative care (5th ed.). Philadelphia: W.B. Saunders, p. 1105. 1570. A client arrives in the emergency room with an eye injury caused by metal fragments that hit the eye while drilling into metal. The nurse checks the eye and notes small pieces of metal floating on the eyeball. Which action would the nurse plan to assist with first? 1. Perform complete visual acuity tests 2. Irrigate the eye with sterile saline 3. Remove the objects using a sterile eye clamp 4. Apply an eye patch Answer: 2 Rationale: Surface foreign bodies are often removed simply by irrigating the eye with sterile normal saline. Clamps would not be used because this action will risk causing further injury to the eye. Applying an eye patch would not provide relief for the problem. Visual acuity tests are not the priority at this time and might not be feasible because the

PN~CD~Questions~1501-1600-

41

client most likely has excessive blinking and tearing as well at this time. Test-Taking Strategy: Use the process of elimination and prioritization skills to answer the question. Focusing on the type of injury sustained and noting the key word floating will direct you to option 2. Review content related to initial treatment of various eye injuries if you had difficulty with this question. Level of Cognitive Ability: Application Client Needs: Physiological Integrity Integrated Process: Nursing Process/Planning Content Area: Adult Health/Eye Reference: Ignatavicius, D., & Workman, M. (2006). Medical-surgical nursing: Critical thinking for collaborative care (5th ed.). Philadelphia: W.B. Saunders, p. 1106. 1571. A client arrives in the emergency room with a chemical eye injury. The nurse immediately: 1. Swabs the eye with an antibiotic ointment 2. Covers the eye with a pressure patch 3. Performs visual acuity tests 4. Irrigates the eye with copious amounts of sterile normal saline Answer: 4 Rationale: Emergency care following a chemical injury to the eye includes irrigating the eye immediately with water, sterile normal saline, or ocular irrigating solution. The irrigation should be maintained for at least 10 minutes. Following this emergency treatment, visual acuity is assessed. Antibiotics and eye patching may be prescribed, but these are not the initial actions. Test-Taking Strategy: Use the process of elimination and prioritization skills to answer the question. Noting the type of injury to the eye and the key word immediately will direct you to option 4. Remember that chemical injuries to the eye require immediate irrigation. Review care to the client who sustained a chemical eye injury if you had difficulty with this question. Level of Cognitive Ability: Application Client Needs: Physiological Integrity Integrated Process: Nursing Process/Implementation Content Area: Adult Health/Eye References: Ignatavicius, D., & Workman, M. (2006). Medical-surgical nursing: Critical thinking for collaborative care (5th ed.). Philadelphia: W.B. Saunders, p. 1106. Lewis, S., Heitkemper, M., & Dirksen, S. (2004). Medical-surgical nursing: Assessment and management of clinical problems (6th ed.). St. Louis: Mosby, p. 445. 1572. A nurse is reviewing the plan of care developed by a nursing student for a client scheduled for keratoplasty. The nurse discusses the plan with the student if which incorrect intervention is listed in the plan? 1. Obtaining a culture and sensitivity with conjunctival swabs 2. Instilling antibiotic ophthalmic medication as prescribed 3. Cutting the client’s eyelashes 4. Administering medications that will dilate the pupil Answer: 4

PN~CD~Questions~1501-1600-

42

Rationale: Keratoplasty is done by removing damaged corneal tissue and replacing it with corneal tissue from a human donor (live or cadaver). Preoperative preparation of the recipient’s eye may include obtaining a culture and sensitivity with conjunctival swabs, instilling antibiotic ophthalmic medication, and cutting the eyelashes. Some ophthalmologists order a medication, such as 2% pilocarpine, to constrict the pupil before surgery. Test-Taking Strategy: Note the key words incorrect intervention in the stem of the question. Focus on the surgical procedure and recalling that the eye is constricted before this surgery will direct you to option 4. Review this preoperative preparation if you had difficulty with this question. Level of Cognitive Ability: Analysis Client Needs: Safe, Effective Care Environment Integrated Process: Teaching/Learning Content Area: Leadership/Management Reference: Christensen, B., & Kockrow, E. (2003). Adult health nursing (4th ed.). St. Louis: Mosby, pp. 582-583. 1573. A nurse is reinforcing discharge instructions to a client following a keratoplasty. Which statement by the client indicates the need for further instructions? 1. “Sutures are removed in 2 weeks.” 2. “I need to avoid lifting heavy objects.” 3. “I need to avoid bending over.” 4. “I need to avoid crowded environments and smoke filled areas.” Answer: 1 Rationale: The client is told that sutures are usually left in place for as long as 6 months. After the sutures are removed and complete healing has occurred, prescription glasses or contact lenses will be prescribed. Options 2, 3, and 4 are correct discharge instructions for the client following keratoplasty. Test-Taking Strategy: Note the key words need for further instructions. These words indicate a false-response question and that you need to select the incorrect client statement. Use the process of elimination recalling that any activities that tend to increase intraocular pressure are avoided. This will assist in eliminating options 2 and 3. Knowing that crowded environments and smoke filled areas increase the chance of inflammation and infection will assist in eliminating option 4. Review client teaching points following keratoplasty if you had difficulty with this question. Level of Cognitive Ability: Comprehension Client Needs: Health Promotion and Maintenance Integrated Process: Teaching/Learning Content Area: Adult Health/Eye Reference: Black, J., & Hawks, J. (2005). Medical-surgical nursing: Clinical management for positive outcomes (7th ed.). Philadelphia: W.B. Saunders, p. 1958. 1574. A nurse is caring for a client following enucleation. On data collection, the nurse notes staining and bleeding on the dressing. The nurse should take which appropriate action?

PN~CD~Questions~1501-1600-

43

1. Document the findings 2. Reinforce the dressing 3. Mark the amount of staining with a black pen and continue to monitor 4. Notify the registered nurse (RN) Answer: 4 Rationale: Postoperative nursing care includes observing the dressing and reporting any staining or bleeding to the surgeon. Options 1, 2, and 3 are inaccurate nursing actions if staining or bleeding is present on the dressing following enucleation. The nurse would notify the RN who would then notify the surgeon immediately. Test-Taking Strategy: Knowledge of the nursing actions related to postoperative complications following enucleation is required to answer this question. Noting the key words staining and bleeding on the dressing will direct you to option 4. If you had difficulty with this question, review postoperative care following this procedure. Level of Cognitive Ability: Application Client Needs: Physiological Integrity Integrated Process: Nursing Process/Implementation Content Area: Adult Health/Eye Reference: Christensen, B., & Kockrow, E. (2003). Adult health nursing (4th ed.). St. Louis: Mosby, p. 582. 1575. A hospitalized client with severe seborrheic dermatitis is receiving treatments of topical glucocorticoid applications followed by the application of an occlusive dressing. The nurse monitors the client for which systemic effect that can occur as a result of this treatment? 1. Adrenal suppression 2. Adrenal hyperactivity 3. Local infection 4. Thinning of the skin Answer: 1 Rationale: Topical glucocorticoids can be absorbed in sufficient amounts to produce systemic toxicity. Primary concerns are growth retardation (in children) and adrenal suppression in all age groups. Systemic toxicity is more likely under extreme conditions, such as with prolonged therapy in which extensive surfaces are treated with high doses of high potency agents in conjunction with occlusive dressings. Test-Taking Strategy: Use the process of elimination. Note the key word systemic in the stem of the question. Options 3 and 4 can be eliminated first because they are local reactions. From the remaining options, knowledge regarding the concerns related to systemic toxicity of topical glucocorticoids will direct you to option 1. Review these systemic effects if you had difficulty with this question. Level of Cognitive Ability: Analysis Client Needs: Physiological Integrity Integrated Process: Nursing Process/Data Collection Content Area: Pharmacology References: McKenry, L., & Salerno, E. (2003). Mosby’s pharmacology in nursing (21st ed.). St. Louis: Mosby, p. 852. Lehne, R. (2004). Pharmacology for nursing care (5th ed.). Philadelphia: W.B. Saunders,

PN~CD~Questions~1501-1600-

44

p. 643. 1576. Isotretinoin (Accutane) is prescribed for a client to treat severe cystic acne. The nurse tells the client that the length of the usual prescribed course of treatment is: 1. 1 month 2. 4 to 8 weeks 3. 15 to 20 weeks 4. 1 year Answer: 3 Rationale: Isotretinoin is usually administered two times daily for a period of 15 to 20 weeks. The usual adult dosage is 0.5 to 1 mg/kg/day. If needed a second course may be administered, but not until 2 months have elapsed after completing the first course. Test-Taking Strategy: Use the process of elimination. Eliminate options 1 and 2 first because the time frames are similar. From the remaining options, select option 3 because a 1-year period is lengthy. Review this medication if you had difficulty with this question. Level of Cognitive Ability: Application Client Needs: Physiological Integrity Integrated Process: Teaching/Learning Content Area: Pharmacology References:  Hodgson, B., & Kizior, R. (2005). Saunders nursing drug handbook 2005. Philadelphia: W.B. Saunders, p. 596. Lehne, R. (2004). Pharmacology for nursing care (5th ed.). Philadelphia: W.B. Saunders, p. 1114. McKenry, L., & Salerno, E. (2003). Mosby’s pharmacology in nursing (21st ed.). St. Louis: Mosby, p. 1133. 1577. A nurse is reinforcing medication instructions to a client with human immunodeficiency virus (HIV) who will be taking saquinavir (Invirase). The nurse instructs the client to take the medication: 1. On an empty stomach 2. Two hours before breakfast 3. Within 2 hours after a full meal 4. At bedtime Answer: 3 Rationale: Saquinavir is an antiviral medication. It is administered within 2 hours after a full meal. If the medication is taken without food in the stomach, it may result in no antiviral activity. Test-Taking Strategy: Use the process of elimination. Note that options 1, 2, and 4 are similar in that they all represent time periods in which the stomach would be empty. Review client instructions regarding the administration of this medication if you had difficulty with this question. Level of Cognitive Ability: Application Client Needs: Physiological Integrity Integrated Process: Teaching/Learning

PN~CD~Questions~1501-1600-

45

Content Area: Pharmacology Reference: Hodgson, B., & Kizior, R. (2005). Saunders nursing drug handbook 2005. Philadelphia: W.B. Saunders, p. 958. 1578. Anastrozole (Arimidex) is prescribed for a postmenopausal client with breast cancer. The nurse assists in developing a plan of care for the client and suggests monitoring the client most closely for which adverse reaction to this medication? 1. Renal failure 2. Thromboembolism 3. Cardiac dysrhythmias 4. Hyperkalemia Answer: 2 Rationale: The most dangerous adverse reaction to anastrozole is thromboembolism. Common reactions include nausea, chest pain, edema, and shortness of breath. A variety of gastrointestinal tract or nervous system effects may also occur. Renal failure, cardiac dysrhythmias, and hyperkalemia are not associated with the use of this medication. Test-Taking Strategy: Specific knowledge regarding this medication is required to answer this question. Recalling that this medication is an antiestrogen and recalling the common concerns related to this classification of medications will direct you to option 2. Review the adverse reactions associated with this medication if you had difficulty with this question. Level of Cognitive Ability: Application Client Needs: Physiological Integrity Integrated Process: Nursing Process/Implementation Content Area: Pharmacology Reference: Hodgson, B., & Kizior, R. (2005). Saunders nursing drug handbook 2005. Philadelphia: W.B. Saunders, p. 74. 1579. A client with acute nonlymphocytic anemia receives treatment with cytarabine (Cytosar-U and ara-C). The nurse reinforces medication instructions to the client and tells the client that it is most important to report which of the following to the physician? 1. Anorexia 2. Nausea 3. Headache 4. Sore throat Answer: 4 Rationale: The major toxic effect of cytarabine is bone marrow depression resulting in hematological toxicity. Signs of hematological toxicity include fever, sore throat, signs of local infection, easy bruising, or unusual bleeding from any site. If these signs occur, the physician is notified. Anorexia, nausea, and a transient headache can occur as side effects of the medication, but do not necessarily warrant physician notification, unless they are persistent in nature. Test-Taking Strategy: Note the key words most important. Recalling that this medication is an antineoplastic and that hematological toxicity can occur with this classification of medications will direct you to option 4. Review the toxic effects of antineoplastic medications if you had difficulty with this question.

PN~CD~Questions~1501-1600-

46

Level of Cognitive Ability: Application Client Needs: Physiological Integrity Integrated Process: Teaching/Learning Content Area: Pharmacology Reference: Hodgson, B., & Kizior, R. (2004). Saunders nursing drug handbook 2004. Philadelphia: W.B. Saunders, p. 278. 1580. Docetaxel (Taxotere) is prescribed for a client with metastatic breast cancer. In addition, dexamethasone (Decadron) is prescribed to be administered before initiation of the docetaxel. The nurse tells the client that the dexamethasone is prescribed to: 1. Enhance the effects of the docetaxel 2. Reduce the severity of fluid retention 3. Prevent thromboembolic disorders 4. Prevent neutropenia Answer: 2 Rationale: Docetaxel is an antineoplastic medication. Frequent side effects include alopecia, hypersensitivity reaction, fluid retention, nausea, vomiting, diarrhea, fever, myalgia, and nail changes. Before receiving docetaxel, the client is premedicated with an oral corticosteroid (dexamethasone 16 mg/day for 5 days beginning day 1 before docetaxel therapy) to reduce the severity of fluid retention or prevent a hypersensitivity reaction. Options 1, 3, and 4 are not actions of dexamethasone. Additionally, dexamethasone is used with caution in a client with thromboembolic disorders. Test-Taking Strategy: Specific knowledge regarding the purpose of administering an oral corticosteroid before treatment with docetaxel is required to answer this question. However, recalling the frequent side effects that occur with the administration of docetaxel will direct you to option 2. Review the procedure related to the administration of docetaxel if you are unfamiliar with it. Level of Cognitive Ability: Application Client Needs: Physiological Integrity Integrated Process: Nursing Process/Implementation Content Area: Pharmacology Reference: Hodgson, B., & Kizior, R. (2005). Saunders nursing drug handbook 2005. Philadelphia: W.B. Saunders, p. 344. 1581. A client with advanced ovarian cancer is being treated with paclitaxel (Taxol). The nurse monitors the client most closely for which side effect of the medication? 1. Bradycardia 2. Constipation 3. Fatigue 4. Hypertension Answer: 1 Rationale: Side effects of paclitaxel include alopecia, pain in the joints and muscles, diarrhea, nausea, vomiting, peripheral neuropathy, hypotension, mucositis, pain and redness at the injection site, cardiac disturbances (bradycardia), and an abnormal electrocardiogram. Fatigue is an occasional side effect. Test-Taking Strategy: Note the key words most closely. Use the ABCs—airway,

PN~CD~Questions~1501-1600-

47

breathing, and circulation—to answer the question. Option 1 represents circulation. Review the frequent side effects associated with the administration of this medication if you had difficulty with this question. Level of Cognitive Ability: Application Client Needs: Physiological Integrity Integrated Process: Nursing Process/Data Collection Content Area: Pharmacology References: Hodgson, B., & Kizior, R. (2005). Saunders nursing drug handbook 2005. Philadelphia: W.B. Saunders, p. 820. Lehne, R. (2004). Pharmacology for nursing care (5th ed.). Philadelphia: W.B. Saunders, p. 1083. 1582. Zidovudine (AZT) is prescribed for an adult client with human immunodeficiency virus. The nurse reinforces instructions to the client about the medication and tells the client: 1. That the medication must be taken with milk 2. To space the medication doses evenly around the clock 3. To discontinue the medication if nausea occurs 4. That aspirin can be taken to treat a headache Answer: 2 Rationale: The adult dosage of zidovudine is 200 mg every 8 hr or 300 mg every 12 hr. The client is instructed to space the doses of the medication evenly around the clock. Food or milk does not affect the gastrointestinal absorption of the medication. The client is instructed to continue therapy for the full length of treatment. The client is also instructed not to take any medication, including aspirin, without the physician’s approval. Test-Taking Strategy: Use the process of elimination. Basic principles related to medication administration will assist in eliminating options 3 and 4. From the remaining options, recalling that this medication is an antiviral will direct you to option 2. Remember that evenly spaced doses are necessary to maintain virustatic concentrations of the medication. Review client teaching points related to this medication if you had difficulty with this question. Level of Cognitive Ability: Application Client Needs: Physiological Integrity Integrated Process: Teaching/Learning Content Area: Pharmacology Reference: Hodgson, B., & Kizior, R. (2005). Saunders nursing drug handbook 2005. Philadelphia: W.B. Saunders, p. 1130. 1583. Stavudine (Zerit) is prescribed for a client with advanced human immunodeficiency virus. The nurse reinforces medication instructions to the client and tells the client that it is most important to report which of the following to the physician? 1. Constipation 2. Headache 3. Numbness or tingling in the hands and feet 4. Loss of energy Answer: 3

PN~CD~Questions~1501-1600-

48

Rationale: Peripheral neuropathy, characterized by numbness, tingling, or pain in the hands or feet, can occur frequently with this medication and is an adverse reaction. Headache, diarrhea (not constipation), and loss of energy are side effects of the medication. Test-Taking Strategy: Note the key words most important. Recalling that peripheral neuropathy is an adverse reaction associated with the use of this medication will direct you to option 3. Review the adverse effects of this medication if you had difficulty with this question. Level of Cognitive Ability: Application Client Needs: Physiological Integrity Integrated Process: Teaching/Learning Content Area: Pharmacology Reference: Hodgson, B., & Kizior, R. (2005). Saunders nursing drug handbook 2005. Philadelphia: W.B. Saunders, p. 987. 1584. A client with human immunodeficiency virus who is taking an oral solution of ritonavir (Norvir) complains about the taste of the solution. The nurse tells the client to: 1. Notify the physician 2. Mix the oral solution with chocolate milk 3. Take the medication at bedtime 4. Refrigerate the solution Answer: 2 Rationale: Ritonavir oral solution is preferably administered with food. It may be mixed with chocolate milk or a dietary supplement to improve the taste. The client is also instructed to consume the dose within 1 hour of mixing. It is not necessary to notify the physician. Taking the medication at bedtime or refrigeration of the medication will not have an effect on the taste of the oral solution. Test-Taking Strategy: Use the process of elimination. Focusing on the issue, the taste of the solution, will assist in eliminating options 1 and 3. From the remaining options, recalling that the medication is preferably administered with food will direct you to option 2. Review client instructions regarding the administration of this oral solution if you had difficulty with this question. Level of Cognitive Ability: Application Client Needs: Physiological Integrity Integrated Process: Teaching/Learning Content Area: Pharmacology Reference: Hodgson, B., & Kizior, R. (2005). Saunders nursing drug handbook 2005. Philadelphia: W.B. Saunders, pp. 944-945. 1585. A nursing instructor is reviewing the nursing care plan prepared by the nursing student who is preparing to care for a client undergoing a pneumonectomy for a lung tumor. Which nursing intervention if documented in the plan of care should the nursing instructor question? 1. Monitor the amount and characteristics of the drainage from the chest tube system 2. Encourage the client to cough and deep breathe 3. Keep the head of the bed elevated

PN~CD~Questions~1501-1600-

49

4. Check the surgical dressing for drainage Answer: 1 Rationale: Closed-chest drainage is not usually used following pneumonectomy. The serous fluid that accumulates in the empty thoracic cavity eventually consolidates. The consolidation prevents shifts of the mediastinum, heart, and remaining lung. The head of the bed is elevated to facilitate adequate oxygenation. Complete lateral positioning is avoided because the mediastinum is no longer held in place on both sides by lung tissue, and extreme turning may cause mediastinal shift and compression of the remaining lung. Options 2, 3, and 4 are general postoperative measures in this surgical procedure. Test-Taking Strategy: Use the process of elimination. Visualize the effects of the surgical procedure in selecting the correct option. Recalling that in a pneumonectomy the entire lung is removed will assist in directing you to option 1. If you had difficulty with this question, review postoperative care following this surgical procedure. Level of Cognitive Ability: Comprehension Client Needs: Physiological Integrity Integrated Process: Teaching/Learning Content Area: Adult Health/Oncology Reference: Christensen, B., & Kockrow, E. (2003). Adult health nursing (4th ed.). St. Louis: Mosby, p. 390. 1586. {PLACE FIGURE HERE (Fig. 15)} Perry, A., & Potter, P. (2002). Clinical nursing skills & techniques (5th ed.). St. Louis: Mosby, p. 891. A nurse is assisting in providing follow-up care to a client with this type of device. The nurse documents that the client is in a: 1. Body jacket cast 2. Gardner-Wells tongs 3. Halo vest 4. Hip spica cast Answer: 3 Rationale: The halo vest is used to treat cervical fractures. The halo vest or jacket has a ring that is fixed to the skull with pins. This ring is then attached to the vest or jacket by rods. This device provides the traction required to maintain cervical alignment and allows early mobilization and rehabilitation. A body jacket cast is applied to the upper torso. Skull tong traction involves the use of one of a variety of tongs (Gardner-Wells, Crutchfield, Vinke, or Barton). These tongs are drilled into the skull or placed below the scalp and attached to ropes, pulleys, or weights. This type of traction is used for cervical vertebrae fractures and involves the use of special beds or turning frames to facilitate nursing care. A hip spica cast is used to treat pelvic and femoral fractures. The cast covers the lower torso and extends to one or both lower extremities. Test-Taking Strategy: Use the process of elimination focusing on the components of this device. Note the relationship between the device and the word “vest” in option 3. If you had difficulty with this question, review this type of device used to treat cervical fractures. Level of Cognitive Ability: Analysis Client Needs: Physiological Integrity

PN~CD~Questions~1501-1600-

50

Integrated Process: Communication and Documentation Content Area: Adult Health/Neurological References: Black, J., & Hawks, J., (2005). Medical-surgical nursing: Clinical management for positive outcomes (7th ed.). Philadelphia: W.B. Saunders, p. 2218. Perry, A., & Potter, P. (2002). Clinical nursing skills & techniques (5th ed.). St. Louis: Mosby, p. 891. 1587. A licensed practical nurse employed in the ambulatory clinic is assisting a registered nurse with preparing to administer an intravenous dose of immune globulin (IGIV). The licensed practical nurse ensures that which medication is readily available before the medication is administered? 1. Protamine sulfate 2. Phytonadione (vitamin K) 3. Epinephrine (Adrenalin) 4. Acetylcysteine (Mucomyst) Answer: 3 Rationale: IGIV is an immune serum that increases antibody titer and antigen-antibody reaction, providing passive immunity against infection. Anaphylactic reactions, although rare, can occur, and the nurse ensures that epinephrine is readily available when administering this medication. Protamine sulfate is the antidote for heparin. Vitamin K is the antidote for oral anticoagulants. Acetylcysteine is used to treat acetaminophen overdose. Test-Taking Strategy: Note the key words readily available. Recalling that an anaphylactic reaction can occur will direct you to option 3. Also use the process of elimination and knowledge of the common antidotes to answer correctly. Review the adverse reactions of IGIV and the nursing considerations related to the administration of this medication if you had difficulty with this question. Level of Cognitive Ability: Application Client Needs: Physiological Integrity Integrated Process: Nursing Process/Implementation Content Area: Pharmacology Reference: Hodgson, B., & Kizior, R. (2005). Saunders nursing drug handbook 2005. Philadelphia: W.B. Saunders, p. 562. 1588. A nurse is reviewing the postoperative orders for a client who has just returned from surgery and notes that the surgeon has prescribed lepirudin (Refludan). The nurse understands that this medication has been prescribed to prevent: 1. Pain 2. Nausea 3. Respiratory complications 4. Thromboembolic complications Answer: 4 Rationale: Lepirudin is an anticoagulant used in clients with heparin-induced thrombocytopenia and associated thromboembolic disease to prevent further thromboembolic complications. In the postoperative client, the initial dose is administered as soon as possible after surgery, but not more than 24 hours after surgery.

PN~CD~Questions~1501-1600-

51

Test-Taking Strategy: Specific knowledge regarding the action of lepirudin is required to answer this question. Recalling that this medication is an anticoagulant will direct you to option 4. Review the action and use of this medication if you had difficulty with this question. Level of Cognitive Ability: Analysis Client Needs: Physiological Integrity Integrated Process: Nursing Process/Planning Content Area: Pharmacology Reference: Hodgson, B., & Kizior, R. (2005). Saunders nursing drug handbook 2005. Philadelphia: W.B. Saunders, p. 620. 1589. Letrozole (Femara) is prescribed for a postmenopausal client with advanced breast cancer. The nurse reinforces instructions to the client regarding the medication and tells the client that the most frequent side effect is: 1. Skeletal pain 2. Weakness 3. Diarrhea 4. Nervousness Answer: 1 Rationale: Letrozole is an aromatase inhibitor that is used to treat advanced breast cancer in postmenopausal women whose disease progressed after antiestrogen therapy. The most frequent side effects include skeletal pain, back, arm, and leg pain. Less frequent side effects include nausea, headache, fatigue, constipation, vomiting, and dyspnea. Test-Taking Strategy: Specific knowledge regarding the side effects of this medication is necessary to answer this question. Remember that the most frequent side effects include skeletal pain, back, arm, and leg pain. If you are unfamiliar with this medication, review the side effects and the associated client instructions. Level of Cognitive Ability: Application Client Needs: Physiological Integrity Integrated Process: Teaching/Learning Content Area: Pharmacology Reference: Hodgson, B., & Kizior, R. (2005). Saunders nursing drug handbook 2005. Philadelphia: W.B. Saunders, p. 622. 1590. A clinic nurse is taking a health history on a client seen in the health care clinic for the first time. When the nurse asks the client about current prescribed medications, the client tells the nurse that amprenavir (Agenerase) is taken twice daily. Based on this finding, the nurse elicits data from the client regarding the presence of which condition? 1. Peptic ulcer disease 2. Inflammatory bowel disease 3. Human immunodeficiency virus (HIV) 4. Coronary artery disease (CAD) Answer: 3 Rationale: Amprenavir is an antiretroviral agent, classified as a protease inhibitor, used to treat HIV infection. It is not used to treat peptic ulcer disease, inflammatory bowel disease, or CAD.

PN~CD~Questions~1501-1600-

52

Test-Taking Strategy: Use the process of elimination. Eliminate options 1 and 2 because they are similar in that they are both conditions of the gastrointestinal tract. From the remaining options, it is necessary to know the classification of this medication. Note the generic name, amprenavir to assist in determining that this medication is used to treat a virus. Review this medication if you had difficulty with this question. Level of Cognitive Ability: Analysis Client Needs: Physiological Integrity Integrated Process: Nursing Process/Data Collection Content Area: Pharmacology Reference: Hodgson, B., & Kizior, R. (2005). Saunders nursing drug handbook 2005. Philadelphia: W.B. Saunders, p. 69. 1591. Capecitabine (Xeloda) is prescribed for a client with metastatic breast cancer. The nurse reinforces information to the client about the medication and tells the client that the most frequent side effect of the medication is: 1. Diarrhea 2. Headache 3. Myalgia 4. Dyspepsia Answer: 1 Rationale: Capecitabine is an antineoplastic medication. Diarrhea is a frequent side effect associated with the medication. Headache, myalgia, and dyspepsia can occur with the use of this medication, but these are not frequent side effects. Test-Taking Strategy: Note the key words most frequent. Use the process of elimination and knowledge regarding this medication to answer the question. Remember that diarrhea is a frequent side effect associated with the medication. Review this medication if you had difficulty with this question. Level of Cognitive Ability: Application Client Needs: Physiological Integrity Integrated Process: Teaching/Learning Content Area: Pharmacology Reference: Hodgson, B., & Kizior, R. (2005). Saunders nursing drug handbook 2005. Philadelphia: W.B. Saunders, p. 160. 1592. Efavirenz (Sustiva), an antiviral medication, is prescribed for a client with human immunodeficiency virus (HIV) infection. The client will be taking a daily dose of 600 mg. The nurse tells the client that it is best to take the medication: 1. In the morning before breakfast 2. At bedtime 3. With lunch 4. With dinner Answer: 2 Rationale: Because the medication causes temporary nervous system side effects during the first 2 to 4 weeks of therapy, the client is instructed to take the medication at bedtime. Test-Taking Strategy: Use the process of elimination. Eliminate options 1, 3, and 4 because they are similar in that they represent time periods that are associated with food

PN~CD~Questions~1501-1600-

53

intake. Review client instructions regarding the administration of this medication if you had difficulty with this medication. Level of Cognitive Ability: Application Client Needs: Physiological Integrity Integrated Process: Nursing Process/Implementation Content Area: Pharmacology Reference: Hodgson, B., & Kizior, R. (2005). Saunders nursing drug handbook 2005. Philadelphia: W.B. Saunders, p. 368. 1593. A male client is admitted to the hospital because of complaints of vomiting and abdominal pain. During data collection, the client tells the nurse that he is taking entacapone (Comtan). Based on this finding, the nurse elicits information from the client regarding the presence of which condition? 1. Hypertension 2. Parkinson’s disease 3. Peripheral vascular disease 4. Hyperlipidemia Answer: 2 Rationale: Entacapone is an antiparkinsonian agent that is used in conjunction with levodopa to improve the quality of life in clients with Parkinson’s disease. It is not used to treat cardiovascular disorders. Test-Taking Strategy: Use the process of elimination. Note that options 1, 3, and 4 are similar in that they all are cardiovascular related disorders. Review the action and use of entacapone if you had difficulty with this question. Level of Cognitive Ability: Analysis Client Needs: Physiological Integrity Integrated Process: Nursing Process/Data Collection Content Area: Pharmacology Reference: Hodgson, B., & Kizior, R. (2005). Saunders nursing drug handbook 2005. Philadelphia: W.B. Saunders, p. 377. 1594. Exemestane (Aromasin), 25 mg orally daily, is prescribed for a client with advanced breast cancer. The nurse reinforces instructions to the client about the medication and tells the client to take the medication: 1. Before breakfast 2. In the morning on arising 3. After a meal 4. Before a meal Answer: 3 Rationale: The best time for a client to take the daily dose of exemestane is after a meal. Options 1, 2, and 4 are incorrect. Test-Taking Strategy: Use the process of elimination. Eliminate options 1, 2, and 4 because they are similar in that they all represent time frames related to medication administration before food intake. If you had difficulty with this question, review its method of administration. Level of Cognitive Ability: Application

PN~CD~Questions~1501-1600-

54

Client Needs: Physiological Integrity Integrated Process: Nursing Process/Implementation Content Area: Pharmacology Reference: Hodgson, B., & Kizior, R. (2005). Saunders nursing drug handbook 2005. Philadelphia: W.B. Saunders, p. 418. 1595. Indinavir (Crixivan) is prescribed for a client with a diagnosis of human immunodeficiency virus (HIV). The nurse reinforces medication instructions and tells the client to: 1. Expect the urine to turn red in color 2. Take the medication with a large meal 3. Expect a significant amount of unexplained weight loss 4. Increase fluid intake to at least 1.5 L/day Answer: 4 Rationale: Indinavir is an antiretroviral agent. This medication can cause kidney stones; therefore the client is instructed to increase fluid intake to at least 1.5 L/day. The client is also instructed to report sharp back pain or the presence of blood in the urine. The client is instructed to take the medication 1 hour before or 2 hours after a large meal. If the medication needs to be taken with food, the client should consume a light meal, such as dry toast, juice, or a bowl of cereal with milk. Unexplained weight loss needs to be reported to the physician. Test-Taking Strategy: Use the process of elimination. Eliminate option 1 because a red color to the urine indicates the presence of blood. Eliminate option 2 because of the word “large” and option 3 because of the word “significant.” Remembering that kidney stones can occur with the use of this medication will direct you to option 4. Review the adverse effects of indinavir if you had difficulty with this question. Level of Cognitive Ability: Application Client Needs: Physiological Integrity Integrated Process: Teaching/Learning Content Area: Pharmacology References: Hodgson, B., & Kizior, R. (2005). Saunders nursing drug handbook 2005. Philadelphia: W.B. Saunders, p. 564. McKenry, L., & Salerno, E. (2003). Mosby’s pharmacology in nursing (21st ed.). St. Louis: Mosby, p. 1036. 1596. Lamivudine (Epivir) is prescribed for a client with human immunodeficiency virus (HIV) who is presently taking zidovudine (Retrovir). The nurse reinforces medication instructions to the client and tells the client: 1. To discontinue the zidovudine during the course of therapy with lamivudine 2. That the medication must be taken with food 3. That numbness of the hands and feet is expected 4. To report the occurrence of vomiting or abdominal pain to the physician Answer: 4 Rationale: Lamivudine is an antiretroviral agent that is administered in combination with zidovudine to delay the appearance of zidovudine resistance. Lamivudine is well absorbed orally with or without food. Peripheral neuropathy can occur with its use, and

PN~CD~Questions~1501-1600-

55

the client is instructed to notify the physician if burning, numbness, or tingling of the hands, arms, feet, or legs occur. Pancreatitis, evidenced by nausea, vomiting, and abdominal pain, is also an adverse reaction to the medication requiring physician notification. Test-Taking Strategy: Use the process of elimination. Eliminate option 1 first using general guidelines related to medication administration. The nurse does not tell a client to discontinue a medication without a specific prescription to do so. Eliminate option 2 next because of the absolute word “must.” From the remaining options, recalling that pancreatitis and peripheral neuropathy are adverse reactions will direct you to option 4. Review client instructions regarding this medication if you had difficulty with this question. Level of Cognitive Ability: Application Client Needs: Physiological Integrity Integrated Process: Teaching/Learning Content Area: Pharmacology References: Hodgson, B., & Kizior, R. (2005). Saunders nursing drug handbook 2005. Philadelphia: W.B. Saunders, p. 614. McKenry, L., & Salerno, E. (2003). Mosby’s pharmacology in nursing (21st ed.). St. Louis: Mosby, p. 1032. 1597. Levalbuterol (Xopenex) via inhalation is prescribed for a client with emphysema. The nurse reinforces instructions to the client regarding the medication and teaches the client about the dietary restrictions that need to be implemented while taking this medication. The nurse determines that the client understands the dietary instructions if the client states to avoid: 1. Bananas 2. Cocoa 3. Orange juice 4. Baked potatoes Answer: 2 Rationale: Levalbuterol is a bronchodilator. This medication stimulates the beta receptors in the lungs, relaxes bronchial smooth muscle, increases vital capacity, and decreases airway resistance. Central nervous system (CNS) stimulation can occur with the use of this medication. The client is instructed to avoid caffeine-containing products, such as coffee, tea, colas, and chocolate, because these products can cause further CNS stimulation. Options 1, 3, and 4 are food items that are high in potassium. Test-Taking Strategy: Use the process of elimination. Noting the client’s diagnosis will assist in recalling that levalbuterol is a bronchodilator. Also, note the key word avoid and the similarity between options 1, 3, and 4 in that they are all food items that are high in potassium. Review the dietary restrictions related to this medication if you had difficulty with this question. Level of Cognitive Ability: Analysis Client Needs: Physiological Integrity Integrated Process: Nursing Process/Evaluation Content Area: Pharmacology Reference: Hodgson, B., & Kizior, R. (2005). Saunders nursing drug handbook 2005.

PN~CD~Questions~1501-1600-

56

Philadelphia: W.B. Saunders, p. 628. 1598. Moxifloxacin (Avelox) is prescribed for a client with community acquired pneumonia. The client needs to take the medication for a period of 10 days, and the nurse reinforces instructions to the client about the medication. Which statement by the client indicates an understanding of the medication instructions? 1. “I need to limit my daily fluid intake.” 2. “I need to take the medication with a magnesium-containing antacid.” 3. “I need to wear sunscreen and protective clothing when outdoors.” 4. “Joint swelling and tendon pain is expected to occur while I am taking the medication.” Answer: 3 Rationale: Moxifloxacin is a fluoroquinolone. Increased sensitivity of the skin to sunlight can occur, and the client is instructed to avoid excessive sunlight and artificial ultraviolet light. The client should wear sunscreen and protective clothing when outdoors. The client should also drink fluids liberally and avoid the use of antacids because antacids will decrease absorption of the medication. The medication can cause inflamed and ruptured tendons, and the client is instructed to notify the physician if inflammation or tendon pain occurs. Test-Taking Strategy: Use the process of elimination. Focusing on the client’s diagnosis will assist in eliminating option 1. General principles related to medication administration will assist in eliminating option 2. From the remaining options, focusing on the key words indicates an understanding in the stem of the question will direct you to option 3. Review client teaching points related to the administration of this medication if you had difficulty with this question. Level of Cognitive Ability: Analysis Client Needs: Physiological Integrity Integrated Process: Nursing Process/Evaluation Content Area: Pharmacology Reference: Hodgson, B., & Kizior, R. (2005). Saunders nursing drug handbook 2005. Philadelphia: W.B. Saunders, p. 736. 1599. An oral powder form of nelfinavir (Viracept) is prescribed for a client with human immunodeficiency virus (HIV). The nurse reinforces instructions regarding the preparation of the medication and tells the client to mix the powder with: 1. Milk 2. Orange juice 3. Applesauce 4. Grapefruit juice Answer: 1 Rationale: Nelfinavir is an antiviral medication used in the treatment of HIV infection when antiretroviral therapy is warranted. It is available in both tablet and powder form. The powder form is prepared by mixing the dose with a small amount of water, milk, formula, soy milk, or dietary supplements. The powder is not mixed with acidic foods or juices, such as apple juice or applesauce, orange juice, or grapefruit juice. Test-Taking Strategy: Use the process of elimination. Eliminate options 2, 3, and 4

PN~CD~Questions~1501-1600-

57

because they are similar in that they are all acidic food items. Review client teaching points related to this medication if you had difficulty with this question. Level of Cognitive Ability: Application Client Needs: Physiological Integrity Integrated Process: Teaching/Learning Content Area: Pharmacology References: Hodgson, B., & Kizior, R. (2005). Saunders nursing drug handbook 2005. Philadelphia: W.B. Saunders, p. 759. McKenry, L., & Salerno, E. (2003). Mosby’s pharmacology in nursing (21st ed.). St. Louis: Mosby, p. 1036. 1600. A nurse is reviewing the physician’s orders for a client scheduled for a cardiac catheterization and notes that the physician has prescribed tirofiban (Aggrastat). The nurse understands that this medication has been prescribed to: 1. Prevent infection 2. Inhibit thrombus formation 3. Prevent dysrhythmias 4. Prevent bleeding Answer: 2 Rationale: Tirofiban is an antiplatelet and antithrombotic medication. It produces rapid inhibition of platelet aggregation by preventing binding of fibrinogen to receptor sites on platelets. This action inhibits thrombus formation. It is used as an adjunct to aspirin and heparin for hospitalized clients at high risk for myocardial infarction or for clients undergoing a cardiac catheterization procedure. Options 1, 3, and 4 do not identify the actions of tirofiban. Bleeding is a side effect of the medication. Test-Taking Strategy: Specific knowledge regarding this medication is required to answer this question. Recalling that this medication is an antiplatelet agent will direct you to option 2. Review the action and use of this medication if you had difficulty with this question. Level of Cognitive Ability: Analysis Client Needs: Physiological Integrity Integrated Process: Nursing Process/Planning Content Area: Pharmacology Reference: Hodgson, B., & Kizior, R. (2005). Saunders nursing drug handbook 2005. Philadelphia: W.B. Saunders, p. 1042.

More Documents from "Linda Kuglarz"

Silvestri Chapter 21 Ed#551
October 2019 28
Silvestri101-200
October 2019 50
Silvestri1301-1400
October 2019 51
Silvestri Chapter 45 Ed#569
October 2019 37
Silvestri Chapter 32 Ed#55c
October 2019 41
Silvestri Chapter 07 Ed#543
October 2019 36